PDA

View Full Version : Theory on lift


Pages : [1] 2

extricate
24th Jul 2012, 16:11
To all aviators,

Need some help here.

I'm trying to understand this concept on lift but i still can't understand the concept when they use flow of air against a moving cylinder. It's regarding the Magnus effect.

I think the whole point of using the cylinder theory is to prove that air flow to the top of the wing has a higher velocity. But how do i visualize it with a moving cylinder and a moving liquid?

Pardon me, this isn't a very technical ques.

Page 3
http://www.faa.gov/library/manuals/aviation/pilot_handbook/media/PHAK%20-%20Chapter%2003.pdf

http://i50.tinypic.com/2ef6qhg.jpg

Lyman
24th Jul 2012, 19:44
The air obviously moves (is accelerated) more rapidly vice the surface in the rotating condition, so the comfortable assumption is to do with friction, and boundary layer? Why am I thinking something to do with Coanda?

Lift? A wing "lifts"? Doesn't it Push? By "fluid" do you mean liquid?

Is "Inviscid" in there anywhere?

awblain
24th Jul 2012, 20:30
The top of the cylinder, moving with the airflow, speeds up the air at any point compared with a non-rotating cylinder, while the bottom of the cylinder, moving against the airflow, slows the air down. Faster air corresponds to lower pressure, slower air to higher pressure, from Bernoulli along the streamline, and so it means there's a net lift force on the cylinder towards the top.

There's also drag pushing/pulling the cylinder in the direction of the flow.

Lonewolf_50
24th Jul 2012, 20:51
Lyman:
Lift? A wing "lifts"?
No, the air lifts the wing via a pressure differential. Vector sums.
Doesn't it Push?
No. A prop pulls, a jet pushes. A wing is the surface area which is pushed by the fluid.
By "fluid" do you mean liquid?
No. Air is a fluid. Smoke is a fluid. Water is a fluid. That said, the diving planes on a submarine use similar flow characteristics to control depth and trim.
Is "Inviscid" in there anywhere?
Is that Spiderman's next arch enemy in the soon to be released Hollywood blockbuster, "Spiderman 5?" :E I'll be the guy near the fire exit in a gas mask ... uh, maybe too soon for that joke. :sad:

boguing
24th Jul 2012, 20:56
It's not you.

It is a very confusing page. There are similarities between rotating cylinders, but it's not helpful to use them for an introduction to aerodynamics.

Probably easiest to google theory of lift on wiki.

And if you become interested in Flettner rotors, do the same for fanwing.

Do not, under any circumstances do the same for helicopters. For they are evil.

Got that JT.

awblain
24th Jul 2012, 21:10
I think Boguing is entirely right.

That picture you posted is wrong - after the air has passed the rotating cylinder its net direction is at a downward angle to the horizontal. There's an upward force on the rotating cylinder, so there's a downward force on the air. The bunching up of the streamlines is weird too: when the flow speed increases, and density remains the same, the transverse separation of the streamlines is less.

john_tullamarine
24th Jul 2012, 22:34
The FAA graphic is a tad useless as it doesn't show the extent of streamline displacement as the cylinder speed cranks up. Indeed, it is more confusing than illuminating in my isolated view.

If one compares the situation with the main field flowing (cylinder off/on) the rotating cylinder effectively constitutes a vortex within the main flow field (don't fuss too much about how you might get a vortex to sit in the middle of a steady flow without being blown away .. mathematics stuff).

The resulting interaction of the two flows is to force the main flow (in the region of the rotating cylinder) to be deflected so that the incoming stream approaches the cylinder from in front and below and departs to the rear and below. This gives a significant change in flow direction to the local main flow and that causes a significant change in momentum .. which provides a vertical force (up/down according to the direction of cylinder rotation) which we call lift (for some strange reason).

The faster the cylinder rotates, the more pronounced is the lift effect.

If you now look at the flow past a wing (say with LE and TE devices), it is pretty similar. This idea then leads to one of the ways of describing lift mathematically using vortex flows.

You might like to have a play with the Jave applet in this NASA page (http://www.physics.org/explorelink.asp?id=4707&q=Magnus effect&currentpage=1&age=0&knowledge=0&item=1) and you'll see the effect far better than I can explain in words.

Easiest way to play is to move the slider bar to the right and watch the spin rate increase, the flow interaction increase, and the lift force increase. The graph to the right of the flow graphic has a little red dot which slides up and down the line to give you an idea of the force developed.

Neat presentation I thought.


I think the whole point of using the cylinder theory is to prove that air flow to the top of the wing has a higher velocity

One of the problems in teaching is trying to match the description to the needs of the person who is trying to come to grips with the problem. Hence we tend to dumb things down so we can emphasise some of the important things happening.

The speed over the top idea is part of the usual way we talk about Bernoulli (and Euler - although he doesn't get any credit in pilot training) flow to emphasise some aspects.

For the cylinder trick, the important thing to emphasise is not the local flow speeds (and your thought is fine - the top flow close to the cylinder will go faster than that on the bottom) but the fact that the spinning cylinder causes the flow to be distorted so that it approaches from front/below and departs to the rear/below.

To me, this is a much more useful way of explaning lift as it gets down to simple action and reaction stuff which we see in lots of places in everyday life.

And, as to the "cylinder theory", it's not a case of its just being theory. If you put a cylinder in a smoke tunnel and spin it up .. the streamline smoke tell tails do JUST what the NASA applet shows.

Figuring out clever ways to change flow direction is all about how you develop desirable forces (lift) in fluids while minimising undesirable forces (drag) .. and this is what aerodynamicists get paid for.

MarkerInbound
24th Jul 2012, 23:24
No. A prop pulls, a jet pushes.

What about a pusher prop?

Lyman
25th Jul 2012, 00:01
Hiya Lonewolf. You must be a helicopter pilot. Twerp the Swiss plumber hisself, Bernoulli, what used the term, Inviscid. He was a contemporary of Bram Stoker, so may heap he coined the term in honor of some bloody beast. He used the term to describe....air. Not solid, no viscosity. He opined that at the velocity of 266.377 knots, air became compressed to serve as a foil on a par with liquids, and solids....I think.

Yes, a wing is acted upon with a push, there is no such thing as a pull in nature, nor is there "vacuum". Differential pressures, SI.

Thrust is what we want, there is no pull, push is what keeps us airborne. Jet, prop, farts, it's all thrust.

Helicopters fly by converting vibration into fear, which levitates the machine and it's contents. Or summat.

RCav8or
25th Jul 2012, 03:18
Quote:
What about a pusher prop?
Good question:p
I guess the prop just produces thrust:O. If the prop has the cranky thing stuck in its backside, then it pulls. If the cranky thing is stuck into its front-side, then it pushes.

Microburst2002
25th Jul 2012, 06:20
The magnus effect, the one that bends the trayectory of a football ball (the round ones, not the other ones, those are rugby) when a football player kicks it in the side, ia used to explain how lift can exist, but you can live without that didactic trick and know what lift is.

Bernouilli is a traditional explanation of lift, and totally unsatisfactory. Newton, however, explains it much better. If the wing pushes the air downwards, the air will push the wing upwards. That's it.

DaveReidUK
25th Jul 2012, 07:00
there is no such thing as a pull in nature, nor is there "vacuum"


Exactly. All this time, Nature has been abhorring something that doesn't actually exist. :O

Radu Poenaru
25th Jul 2012, 07:22
No. A prop pulls, a jet pushes. A wing is the surface area which is pushed by the fluid.


Actually, you can consider the prop blade to be a wing. In which case "A wing/propeller is the surface area which is pushed by the fluid."

Using a rotating cylinder to explain lift is a difficult approach in the first place, and the images you have attached are doing a poor job in detailing what is really going on there.

Checkboard
25th Jul 2012, 08:00
Here's a technology demonstrator displaying the concept as an actual flying model:

FanWing a new type of aircraft - YouTube

Here's a more in-depth article:

Fan Wing - YouTube

... I'd hate to see the glide ratio if the engine stops :uhoh:

FlightPathOBN
25th Jul 2012, 15:11
Bernoulli's principle is generally explained by the argument that the faster speed of the air along the top of the wing leads to reduced air pressure above and hence produces a lift.
Given this, simplistically, one would rationally surmise that the fuselage negates this basic couple.

If we look at Bernoulli's Laws, or what is the common explanation of wing aerodynamics. Airfoils are curved on top and flat below, and therefore the air follows a longer path above than below. Since the upper surface of the wing is longer, it causes the upper air to flow faster than the lower, which (by Bernoulli's principle) creates lower pressure above.
According to this pure principle, a wing can create lift at zero attack angle, and do not deflect air, the air behind the wing is flowing the same as the air ahead.
Are there any examples of the wing section at zero attack angle providing lift?

https://encrypted-tbn1.google.com/images?q=tbn:ANd9GcQK-mlPrVAqnn3T8nm1gmFInQVwDXbjoR-XnmslUIv2uCJfdDmZSw

We have all seen the diagrams, with the airflow coming together nicely at the back of the wing, in a straight line with the air from below.

To add further issue, given what we have seen in wake vortex creation, does that seem plausible?

https://encrypted-tbn0.google.com/images?q=tbn:ANd9GcTUuIiJYBP875w5Nq1Ppe6mHaatG222ARgvm1do2-QJZ0NhepDZNA

Then again,
If we look at Newtons Laws, using principles of Newtonian Angle, wings are forced upwards because they are tilted and they deflect air. The air behind the wing is flowing downwards, while the air far ahead of the wing is not.
Both the upper and lower surfaces of the wing act to deflect the air.
The upper surface deflects air downwards because the airflow "sticks" to the wing surface and follows the tilted wing, called the "Coanda Effect" (marine thrusters and ducted fan UAV's).
(note: while flaps radically effect lift, they add no surface length over the wing, this appears counter to Bernoulli)
For this to be applicable, air's inertia is critical, so after the wing has passed by, air must remain flowing downwards...sound familiar, ie wake vortex creation?

Newtonian physics also explains the lift generated by the center wing section, while Bernoulli does not..


Given that....

Newton and Bernoulli do not contradict each other. Newton's Laws based on air deflection explain 100% of the lifting force. Bernoulli's Laws based on air velocity also explain 100% of the lifting force.

awblain
25th Jul 2012, 20:41
Bernoulli doesn't say anything about where the streamlines go, since it has no description of momentum conservation, although it does a good job of describing what happens along them, since it's describing energy conservation.

Newton deals with momentum.

You need both conservation laws to describe the full picture. It's also hard to picture lift without dealing with drag at the same time, as the air is slowed/made turbulent and deflected downward behind the lifted object.

fantom
25th Jul 2012, 20:46
You are all mad; I am the only sane one.

Type1106
25th Jul 2012, 20:53
I'll bet 'extricate' wishes he'd never started this!:sad:

Lonewolf_50
25th Jul 2012, 20:57
OK, Marker, your lesser case is different from the general case, and was IIRC what Wilbur and Orville used in their early craft. Like ship propellers, eh? :ok:

Lyman:

Helicopters fly by converting vibration into fear, which levitates the machine and it's contents. Or summat.

Since we helicopter pilots typically combine vibration and thrust, the ladies find us very popular. :E

As to "helicopters are evil" I'll suggest to JT that there is at least a grain of truth in that, given the infamous "helicopter pilots are different" piece by Harry R, and the generally accepted 10X cost per pound gross weight to build a helo versus fixed wing. Such is the price we pay for hovering, or being able to fly backwards, which is a good thing, albeit expensive.

Further for aerodynamicists, the complex interactions of lift production and its byproducts on the multiple rotating wings of a helicopter is an ever fascinating subject, but likely beyond the scope of this thread.

All that considered, it's still a wing with a fluid flowing over it.

Lyman
25th Jul 2012, 21:09
As is McD NOTAR...

There are elements of Coanda, Blown Flap, and Bernoulli abounding....

As to Bernoulli, after staring at the ugly leading edge of the P-80, I tested an exaggerated camber wing. The upshot is, without AoA Bernoulli sucks eggs, and Newton rules. The difficult proposition is to mathematically calculate "o" AoA.

One cannot introduce enough airstream onto the leading edge to counter the equal split of drag at O AoA. The lifting force at 0 AoA is parallel the chordwise neutral line, and in opposition to the airstream.

FlightPathOBN
25th Jul 2012, 21:19
now you understand why winglets look like they do....

henra
25th Jul 2012, 21:46
The funny thing is that both theories (Newton and Bernoulli) are indirectly linked to each other.
Newton assumes that air behind the wing is accelerated downward. If you follow the air movement further downstream at one point the diretion of flow will be longitudinal in line with free stream (vortices aside).
The vertical distance between the TE and this point will a) relate to the vertical component of the mass volume flow (Newton) and b) relate to the horizontal volume flow and thus the acceleration of the air above the wing (Bernoulli).

Edit: It is worth looking at different wing polars to cross check. E.g. NACA4506 vs. 2Rz12.
The former being a very thin profile with cl > 0,3 at Alpha = 0°
The latter being a rather thick profile with cl < 0,05 at Alpha = 0°.

The accleration of the air above the wing does not only depend on the thickness of the profile alone but rather on the entire 'expansion' of the flow also behind the wing itself. And that is where Bernoulli will meet Newton.

italia458
26th Jul 2012, 00:23
Lift is very complicated. Nobody currently knows why a wing can create lift. It's similar to the fact that nobody currently knows why there is inertia. We have detailed theories and laws where we can accurately predict how inertia will affect an object and we know inertia is proportional to mass but we don't know why. To get the best understanding of lift I've found that you need to understand as many theories regarding lift as you can. All the theories tell the story of lift that's not complete and all from different angles. When you analyze lift from all these different angles you start to build a pretty decent understanding of lift. I'd recommend getting the basic understanding of all the theories (or as many as you can understand) and then keep going over them, and every time that you do go over them you should be able to go further in depth.

Regarding the rotating cylinder: it's showing that a rotating cylinder has a similar effect as an airfoil does. They both produce lift, they both increase the speed of flow over the top and decrease the flow over the bottom. There is upwash and downwash. This will tie into Kelvin's circulation theorem. Eventually this Magnus effect should seem somewhat intuitive. Magnus effect - Wikipedia, the free encyclopedia (http://en.wikipedia.org/wiki/Magnus_effect)

John Tulla posted a nice explanation here too.

No. A prop pulls, a jet pushes. A wing is the surface area which is pushed by the fluid.

That's hard to define. I think it's easy to see that a jet pushes as it accelerates a flow of air behind it at a higher velocity. But a propeller actually pulls 50% and pushes 50%. Consider a prop that accelerates the local flow velocity by 100 m/s. There will be a 50 m/s increase by the time the air reaches the propeller and the last 50 m/s increase will happen somewhere down stream of the propeller. So with either a Cessna 182 or a Piaggio Avanti II, they both pull AND push.

http://www.gidb.itu.edu.tr/staff/emin/Lectures/Ship_Hydro/SHIP%20HYDRODYNAMICS_LECTURE_NOTES_PART_5_PROPELLER_THEORIES .pdf

Equation 13 and the paragraph below that show this. I like that document as the way lift is created for an airplane is the same way thrust is created for a ship. A different way of looking at it might make it clearer. The document is orientated more towards the physics inclined person.

Lyman
26th Jul 2012, 00:40
Hi Italian....

"That's hard to define. I think it's easy to see that a jet pushes as it accelerates a flow of air behind it at a higher velocity. But a propeller actually pulls 50% and pushes 50%. Consider a prop that accelerates the local flow velocity by 100 m/s. There will be a 50 m/s increase by the time the air reaches the propeller and the last 50 m/s increase will happen somewhere down stream of the propeller. So with either a Cessna 182 or a Piaggio Avanti II, they both pull AND push. "

A turbojet does the same push/pull...

In extreme cases, the intake can produce well over half of the thrust of the exhaust.

In such a case, the exhaust can be redirected to flow against the a/c forwards, and the aircraft will still fly. A jet engine is full of propellors

Put propellers in a tube, and spin them, will there be thrust? Yes.

Power these propellers with an engine that drives them mechanically, still, thrust.

Inject fuel behind the propellers, and ignite it. Thrust.

italia458
26th Jul 2012, 01:10
Lyman...

A jet engine's blades aren't there to propel the airplane forward (assuming we're talking strictly about a turbojet). They're called compressor blades because they compress the air then fuel is added and the air is ignited. The expansion of the fuel/air mixture is directed out the rear of the engine which produces the thrust that propels the airplane forward. A turbojet is really not related to a propeller.

Engine Pressure Variation - EPR (http://www.grc.nasa.gov/WWW/k-12/airplane/epr.html)

In extreme cases, the intake can produce well over half of the thrust of the exhaust.

Do you have a reference for that on a turbojet engine? - specifically where it says that it produces "well over half the thrust".

I'm assuming you're talking about ram effect. There can be a noticeable increase in thrust due to the compression of the air prior to entering the engine (ram effect) if the airplane is traveling at high speeds. But I'm unsure what you're trying to point out.

Lyman
26th Jul 2012, 01:35
I am proposing negative ram. The tailpipe is dramatic, but the gas comes from somewhere, in the case of the over square intake, cool air comprises more mass than the exhaust. If you could paint the exhaust red, and the intake blue, the huge blue cone in front of the engine is lower in pressure than the small red cone in the back. The aircraft is falling forward into its own 'blue hole', and the front of the intake disc is massively more involved in propulsion than the doughnuts on a rope in back.

Pratt/Whitney. J58.

italia458
26th Jul 2012, 02:26
Regarding the J58:

At Mach 3.2, 80% of the engine's thrust came from the ramjet section, with the turbojet section providing 20%.

A ramjet is not the same as "ram effect" on a turbojet engine. You will not get even near half the thrust from ram effect on a turbojet.

john_tullamarine
26th Jul 2012, 02:45
specifically where it says that it produces "well over half the thrust".

If one looks throughout an engine, we get a bunch of pressure variation. Integrating over the various surfaces lets us express that in terms of normal forces. The observation is that, in a well designed installation, a large proportion of the nett forward thrust can be achieved from surface pressure contributions.

As to proportions, this will vary between installations.

The Concorde thread had some discussion on the subject so far as it relates to the Olympus installation and may be worth a read.

This page (http://www.concordesst.com/powerplant.html) suggests that the Concorde had a prodigious proportion of non-engine thrust contribution in supersonic flight .. something in excess of 90%.

... standard engine installation design consideration, regardless of which OEM and model.

Lyman
26th Jul 2012, 02:51
Hi Italian

"The turbojet components of the engine thus provide far less thrust, and the Blackbird flies with 80% of its thrust generated by the air that bypassed the majority of the turbomachinery undergoing combustion in the afterburner portion and generating thrust as it expands out through the nozzle and from the compression of the air acting on the rear surfaces of the spikes."

To be precise, the J58 is at no time a "ramjet". The 80% thrust referenced above is not combusted air, but intake air, isolated from the turbo machinery and the after burner. It has more in common with "high bypass" turbofans in this mode than with a pure ramjet, a device that has no internal compressor section, only ignition. It is in this regard that the division of pressure at the intake cone resembles the propellor, though the zones are defined more by geography within the ducting than the metal stators, due the intense differentials.

IMO. What say you?

pattern_is_full
26th Jul 2012, 04:36
As to whether a given engine "pushes" or "pulls" - simply put a stressmeter on the connection between engine and airframe and determine whether the stress is in tension ("pulls") or compression ("pushes.")

You can fly with a barn door for a wing - with enough power. Water skis provide lift with little or no Bernoulli input. Just downwash and AoA.

However, Bernoulli effect makes the process much more efficient, and makes the difference between being able to fly, and being able to fly functionally (with a useful amount of payload capacity and range).

With barn doors for wings, Orville and Wilbur would have needed closer to 1,200 hp to get off the ground, rather than 12.

italia458
26th Jul 2012, 04:47
John...

I should have been more clear. I was talking about ram effect on a basic turbojet (subsonic) engine which would provide a slight rise in compression at the entrance to the engine which would produce more thrust as it went through the burner stage.

The Concorde engine is quite a bit different than a basic turbojet. I don't fully understand the engine but it's quite interesting! I'll have to read through some of those Concorde threads... there are some very knowledgable people on here.

Lyman...

To be precise, the J58 is at no time a "ramjet".

Reference this: Ramjet - Wikipedia, the free encyclopedia (http://en.wikipedia.org/wiki/Ramjet)

You will see this quote: "The SR-71's Pratt & Whitney J58 engines act as turbojet-assisted ramjets at high-speeds (Mach 3.2)."

Another reference: Factsheets : J58 Turbojet Engine (http://www.hill.af.mil/library/factsheets/factsheet.asp?id=5786)

It's a turbo-ramjet. A ramjet won't function when there is no airflow. The turbojet is what propels the airplane up to speeds that will have the ramjet part of the engine function. The J58 is essentially a turbojet inside a ramjet.

The 80% thrust referenced above is not combusted air, but intake air, isolated from the turbo machinery and the after burner.

Not what I've read. That hot, compressed intake air is fed to the afterburner section.

john_tullamarine
26th Jul 2012, 05:49
I was talking about ram effect

Understood .. but that is a secondary consideration.

The end result of the pressure distribution acting on ancilliary structure can produce a significant force on that structure. Include some clever design .. and one gets a lunch heavily subsidised by Mother Nature.

CliveL, djpil (and, no doubt, others whose specific backgrounds I am not familiar with) being experienced aerodynamicists .. can speak on such things for ever and a day, I'm sure.

Owain Glyndwr
26th Jul 2012, 05:56
For an authentic explanation of how lift is developed check this one out:
The Origins of Lift (http://www.arvelgentry.com/origins_of_lift.htm)

It is written around sails, but the explanation is equally valid for wings. Arvel Gentry is a practical professional aerodynamicist so can be believed. After 40 years of accepting the "Bernouilli" explanation he certainly opened my eyes!

PS: the Olympus 593 (Concorde engine) is a pure conventional turbojet and the "thrust" coming from the intake at Mach 2.0 was about 75% of the total.

italia458
26th Jul 2012, 06:18
Owain...

Excellent article! There is a lot more to lift than Bernoulli.

Back to thrust produced by the intake...

Is there a good article that explains how this all works?

My understanding is that the air is compressed by the intake as the intake slows the air to subsonic speeds. The air is heated as a result of the compression. The resulting air is directed around the 'core' of the engine and put into the afterburner section where fuel is added to the hot air and ignited, producing significant thrust.

I had read that the air gets heated to a very high temperature and was led to believe that if fuel was introduced to the air (in the afterburner section), it would auto-ignite. Is that correct?

Since it's the intake that compresses the air that gets burned and produces 63% of the thrust on the Concorde, they say "the intake produces 63% of the thrust". Is that correct?

Microburst2002
26th Jul 2012, 06:25
Newton is the man.

All you have to find is what does the air do when a wing passes through it. It accelerates the downwards and forward, so the air gives the wing an equal force upwards and backwards.

The more air there is, the more lift (either by increased density, wing surface or angle of attack)
the faster the air is, the more air is accelerated,
the faster the air is, the more accelerated it is

hence, lift is proportional to density, wing surface and angle of attack, and to squared speed. then you use the Lift Coefficient as the constant of proportionality, only it depends on the angle of attack, and that is the formula

john_tullamarine
26th Jul 2012, 06:58
Is there a good article that explains how this all works?

Really only necessary if you need to be able to run some calculations for design work.

To get a general overview idea, think ...

(a) structure exists .. and we are looking more at the region around and outside the engine itself but still subject to airflow due to the engine.
(b) air is flowing over the structure due to the engine's good work and the aircraft's flight speed
(c) air has pressure which varies with local speed
(d) hence the air over different bits of structure will have slightly different pressure
(e) for each little bit of structure, the force associated with the air pressure acts normal (perpendicular) to the little bit of structure
(f) resolve the little bits of force into whatever directions are of interest (ie typically forward and aftwards are what we are after)
(g) integrate (add up) all the little bits of contributing force
(h) end result gives a nett thrust or drag
(i) the thrust or drag varies through the engine so, in some regions, we are getting useful thrust (nacelle and nozzle) while, in others, it's all drag.

One needs to keep in mind that this is the province of the aeroplane designers rather than the engine folk. The engine has to hang inside some structure and have suitable airflow conditions presented at the front and somewhere for the hot bits to get out at the back.

A similar argument can be structured for what happens inside the engine proper.

Owain Glyndwr
26th Jul 2012, 06:59
@Microburst

I'm not going to disagree with your overall position that Newton's laws can be used to calculate lift, but that is not quite the same as saying that they explain how lift is generated. If I may take an extract from Gentry's article:

Note that in this entire discussion I have not once mentioned anything about (1) the air having farther to go on the the top side of an airfoil, or (2) Newton’s laws of motion, or (3) about getting lift by “deflecting the air downward”. In the first case, there is nothing in aerodynamics requiring the top and bottom flows having to reach the trailing edge at the same time. This idea is a completely erroneous explanation for lift. The flow on top gets to the trailing edge long before the flow on the bottom because of the circulation flow field.
As for Newton, his laws are included within the aerodynamic theories discussed.
And on the "deflecting the air downward" idea, that is a three-dimensional effect. In our 2-D case, the circulation flow field causes the air out in front of the airfoil to be directed upward around the airfoil and then back down to about the same level as it started out in front. Yet due to viscous effects and resulting circulation, lift is generated. Yes, we can't fly with a two-dimensional wing and, therefore, are influenced by three dimensional effects caused by a complex trailing vortex system. We can reduce these 3-D effects by using very long wings such as on gliders or the around the world aircraft design by Bert Ruttan. On an infinitely long wing, the 3-D effects are gone and we are essentially back to looking at two-dimensional airfoil aerodynamics. If we can reduce the 3-D effects, then "deflecting the air downward" is not essential to the origins of lift.

WRT your specific remarks:


All you have to find is what does the air do when a wing passes through it. It accelerates the downwards and forward, so the air gives the wing an equal force upwards and backwards.
Not sure about that acceleration forward bit :(
The more air there is, the more lift (either by increased density, wing surface or angle of attack) OK
the faster the air is, the more air is accelerated, OK
the faster the air is, the more accelerated it is
Why would that be?

Owain Glyndwr
26th Jul 2012, 07:24
@Italia

My understanding is that the air is compressed by the intake as the intake slows the air to subsonic speeds. The air is heated as a result of the compression. The resulting air is directed around the 'core' of the engine and put into the afterburner section where fuel is added to the hot air and ignited, producing significant thrust.


This is a turbojet we are talking about - there is only the "core", no fan. Fuel is not put into the "afterburner" - at least not in cruise. It goes into combustion chambers aft of the compressor and before the turbine. The 'engine' contribution to thrust is carried on the turbine blades which, contrary to one of your earlier posts, behave very much like a multi-stage propeller.

I had read that the air gets heated to a very high temperature and was led to believe that if fuel was introduced to the air (in the afterburner section), it would auto-ignite. Is that correct?


I dunno, but since the afterburner sits behind the turbine (which has a very high exit temperature anyway) the question has no real meaning

Since it's the intake that compresses the air that gets burned and produces 63% of the thrust on the Concorde, they say "the intake produces 63% of the thrust". Is that correct

Picking up on JT's comments, the intake on a supersonic aircraft is essentially a convergent/divergent passage. In the first bit (convergent) the air is slowed down from freestream speeds to Mach 1.0. In the divergent bit it is slowed from Mach 1.0 to about 0.6M or whatever the engine can accept. Over the front bit the compression generates pressure forces on forward facing surfaces and gives drag. In the subsonic divergence the pressure increases steadily going towards the engine. This part of the intake has aft facing surfaces and the increased pressures generate thrust. Whether you get a net drag or thrust depends on the Mach number and the intake design - hence Brian Abraham's comments.

The Concorde thread is a good source of information on this.

italia458
26th Jul 2012, 07:48
Owain...

This is a turbojet we are talking about - there is only the "core", no fan. Fuel is not put into the "afterburner" - at least not in cruise. It goes into combustion chambers aft of the compressor and before the turbine. The 'engine' contribution to thrust is carried on the turbine blades which, contrary to one of your earlier posts, behave very much like a multi-stage propeller.

I wasn't talking about a turbojet, I was asking about the intake system that creates 63% of the thrust at cruise speed.

Regarding the multi-stage propeller bit: I understand that the blades act like a multi-stage propeller but I was saying their purpose is to compress the air and not to directly provide thrust like a propeller.

This image shows the Concorde's engines while in supersonic flight.

http://www.concordesst.com/graphics/engineairflow2.jpg

What is the "inlet air"? Where does that "inlet air" go? In the picture it appears that some air goes along the top, following the first arrow and bypassing the engine intake, through a tiny passage and then gets dumped at the rear of the engine right before the divergent exit. There is also another passage on the bottom of the engine where air bypasses the engine intake and meets at the rear of the engine right before the divergent exit. Does the air that exits out of those two passages contribute to the "inlet thrust"?

Edit: 8% of the thrust is from the engine in cruise - does that mean that the fuel burned is only related to that 8% that is from the engine? NONE of the thrust from the inlet is made by combustion?

Easy Street
26th Jul 2012, 08:18
The 'engine' contribution to thrust is carried on the turbine blades which, contrary to one of your earlier posts, behave very much like a multi-stage propeller.

The turbine removes energy from the exhaust gases in order to power the compressor. In no way does the turbine provide thrust, so the propeller analogy is wrong!

Owain Glyndwr
26th Jul 2012, 08:30
I wasn't talking about a turbojet, I was asking about the intake system that creates 63% of the thrust at cruise speed.

OK, you are asking about the bleed air, but it wasn't clear from the context of your text.

Regarding the multi-stage propeller bit: I understand that the blades act like a multi-stage propeller but I was saying their purpose is to compress the air and not to directly provide thrust like a propeller.

True of the compressor blades (on a turbojet) but not true of the turbine blades which do provide thrust (reaction to the acceleration of exhaust gases), just like a propeller produces a higher than freestream velocity behind it.

What is the "inlet air"? Where does that "inlet air" go? In the picture it appears that some air goes along the top, following the first arrow and bypassing the engine intake, through a tiny passage and then gets dumped at the rear of the engine right before the divergent exit. There is also another passage on the bottom of the engine where air bypasses the engine intake and meets at the rear of the engine right before the divergent exit. Does the air that exits out of those two passages contribute to the "inlet thrust"?

From what I know of the design, the "inlet air" is everything that went into the mouth of the intake. Some of that was bled off the roof of the intake (about where the flow went through Mach 1.0. That bleed air was, I believe, ducted to flow around the outside of the engine as cooling air. The top and bottom passages you refer to are two bits of the same passage - it was just split to get cooling air to the lower part of the engine.

This cooling air has to be dumped as efficiently as possible - on Concorde this was done by using it as 'secondary flow' in the final propelling nozzle. It therefore contributes to thrust eventually, but would not be considered to be part of intake thrust as being discussed here.

Edit: 8% of the thrust is from the engine in cruise - does that mean that the fuel burned is only related to that 8% that is from the engine? NONE of the thrust from the inlet is made by combustion?

Not sure what you are getting at, but certainly none of the intake thrust was made by combustion - unless you count that bit of the combustion energy that was used to drive "the inlet pumps" as described earlier!

Owain Glyndwr
26th Jul 2012, 08:35
The turbine removes energy from the exhaust gases in order to power the compressor. In no way does the turbine provide thrust, so the propeller analogy is wrong!I beg to differ!. Sure the turbine removes energy to drive the compressor, but if that was all it did there wouldn't be any thrust. The whole essence of a jet engine, at least as I understand it, is that the machine accelerates the air going in at the front to give it increased momentum when it leaves the engine. From what part (of a pure turbojet) does it get that increased velocity if it isn't the turbine?

To quote from Rolls Royce "the jet engine"

From C to D (expansion through turbine and nozzle on a working cycle chart) the gases resulting from combustion expand through the turbine and jet pipe back to atmosphere. During this part of the cycle some of the energy in the expanding gases is turned into mechanical power by the turbine and the remainder, on its discharge to atmosphere, provides the propulsive jet.

Italics are my explanatory addition, bold is my emphasis

keith williams
26th Jul 2012, 08:59
This link was taken from the Rolls Royce book "The Jet Engine". It shows where the thrust is produced in subsonic flight.

http://www.pprune.org/spectators-balcony-spotters-corner/486887-forward-acting-thrust.html#post7234049

Owain Glyndwr
26th Jul 2012, 09:07
I stand corrected!

I never thought so much of the thrust was developed in the combustion chambers. :O

john_tullamarine
26th Jul 2012, 09:24
.. and the thrust so described is due to the application of gas pressures to the abutting structure.

The graphic gives the story for one engine ie internal story.

Now one has to look at what is happening in the intake system forward of the engine inlet face and the exhaust ducting aft of the turbine.

With a supersonic aeroplane (particularly) there is a tremendous amount of energy in the incoming airflow. As Owain observed earlier, the engine can't accept this supersonic airflow. We have a quite critically designed and controlled set of gadgets to provide a tightly managed system of oblique (ie the shock wave is inclined rather than normal to the direction of flow - such shocks are better for energy losses) shocks (shockwaves) ending up with the flow going through a final relatively weak normal (perpendicular) shock wave before it is massaged onto the forward face of the engine compressor inlet.

The engine itself is a comparatively highly strung thoroughbred stallion and doesn't take kindly to operating conditions which are too far off the design operating conditions case .. hence the various means of dumping excess air to keep the horse happy. With a bit of careful design, the varying air pressure can be managed to create a nett forward thrust on the whole of the inlet and outlet system.

Hence the figures which have been quoted indicating that, at high supersonic speeds (especially), the engine contributes only a small proportion of the total thrust pushing the aeroplane along ...

italia458
26th Jul 2012, 09:29
Owain...

True of the compressor blades (on a turbojet) but not true of the turbine blades which do provide thrust (reaction to the acceleration of exhaust gases), just like a propeller produces a higher than freestream velocity behind it.

I agree with Easy Street. I misread and assumed you were talking about compressor blades. The turbines are extracting energy from the airflow - similar to the air rotating the propeller when you have an engine failure. It produces lots of drag.

Not sure what you are getting at, but certainly none of the intake thrust was made by combustion - unless you count that bit of the combustion energy that was used to drive "the inlet pumps" as described earlier!

Let's say the airplane is cruising at altitude at lower speeds and 100% of the thrust is from the turbojet. Nothing from the inlet or nozzle. The thrust is 100,000 lbs total and is consuming 100 gal/hr. Now, let's speed up to the normal cruise where the turbojet is producing 8% of the total thrust. Let's say the thrust is 200,000 lbs for normal cruise. 8% is 16,000 lbs and the corresponding fuel flow would be 16 gal/hr for that thrust. So, does this mean that only 16 gal/hr are leaving the fuel tanks when the airplane is in cruise?!

It seems ludicrous that you can get 92% thrust for free! I'm trying to understand what is going on here.

italia458
26th Jul 2012, 09:39
John...

I understand what you said about managing intake air so that the engine only receives subsonic air by controlling the shockwaves but why can't the engine accept supersonic air? Is it because of the shockwaves that would develop as it went through the compressor section which would severely disrupt the airflow on the compressor blades?

I understand how a divergent duct can take sonic air and increase it to supersonic but I'm still not quite clear on the intake part and how it contributes to thrust.

italia458
26th Jul 2012, 09:51
Just came across this. It's a bit elementary but quite good I think for getting an idea of what's going on.

Rolls-Royce: Journey Through A Jet Engine (http://www.rolls-royce.com/interactive_games/journey02/flash.html)

rudderrudderrat
26th Jul 2012, 10:37
Hi italia458,
It seems ludicrous that you can get 92% thrust for free! I'm trying to understand what is going on here.
Have a look at M2dude's post at
http://www.pprune.org/tech-log/426900-concorde-engine-intake-thrust.html - the total "thrust" is not for free. The figures mentioned are the proportions of thrust transferred to the airframe by the individual components.

john_tullamarine
26th Jul 2012, 11:05
why can't the engine accept supersonic air?

In subsonic flow, the pressure waves moving upstream from an approaching body have time to cause the air to get out of the road, as it were, without too much disruption ..

However, in supersonic flow, the approaching body comes on the scene without an introduction and the flow, in deflecting abruptly, generates shocks. These are undesirable due to energy losses but can't be avoided. Hence the effort which goes into designing intakes to function as efficiently as practicable.

Now, if the incoming flow to the compressor were to be supersonic, the interactions would be complex to the point of chaotic.

The Concorde threads have plenty of commentary on intake design and might be worth a read.

keith williams
26th Jul 2012, 11:52
I understand how a divergent duct can take sonic air and increase it to supersonic but I'm still not quite clear on the intake part and how it contributes to thrust.


As JT has explained, supersonic intakes employ a series of shock waves to decelerate the incoming air to subsonic speed. This converts most of the dynamic pressure into static pressure. The final part of the intake is a divergent duct, so as the (now subsonic) air flows through it, the deceleration process continues, causing a further increase in static pressure.

The overall result is that the static pressure inside the intake is much greater than that outside the intake. This high static pressure inside the intake pushes forward against the divergent sides, thereby exerting a forward force on the intake.

Provided the engine keeps running at a sufficiently high RPM it will draw the air out of the rear of the intake. This prevents the high static pressure from exerting a rearward force on the aircraft and also enables the air to continue to flow into the intake.

Shutting down the engine would remove the pumping effect. This would cause the airflow through the intake to break down, producing a very large pressure rise in the intake. The intake would then stop producing thrust and start producing a great deal of drag.

Lyman
26th Jul 2012, 13:43
Italian

Hi. I consider both compressor and turbine to be propellers. One drives, the other is driven. The turbine can be considered to be windmilling.

I have a simple brain.

Lyman
26th Jul 2012, 13:46
My stupid spell check prevents me from spelling your name correctly.

Italia

Microburst2002
26th Jul 2012, 14:36
Hi Owain

It is true that Newton does not really explain Lift, but it is a much less generic way of looking at it than Bernouilli, same as Newton laws give much more detail about a body going downslope than energy conservation law does.

Darrol Stinton has a very nice explanation wich put the focus on the air particles rather than the wing. That is the best I have ever seen because the order in which events takes place is the correct one, in my opinion. That is: the wing pushes the air, the air is not easy to be compressed as a result, but pressure changes will create pressure gradients all around the wing, and the air will flow from higher to lower pressure, and the resulting motion, if we look at Newton third law, reveals the existence of Lift and Drag. According to this, the pressure changes come first and the speed changes are a result, and not the other way round, as bernouilli's explanation suggests.

the faster the air is, the more accelerated it is
Why would that be?

Well, this one I must confess I am making an assumption. That assumption is that for a given set of conditions, the speed imparted to the air is a "given percentage". So the faster the air stream is, for that same "percentage" means that the air is imparted a higher speed. Something like: if airspeed is 100 kt, the air will be accelerated 10 kt (a 10%). If the airspeed is 200 kt, then the air will be accelerated 20 kt. Double acceleration, double force.

If I am not right, then there must be another reason why increasing speed increases lift, aside from more air being "processed" in a given instant. Otherwise speed wouldn't go squared.

I also hate the "theory" of the faster molecule above and the slower one below to reach the trailing edge at the same time, and I hate Koanda even more.

Lyman
26th Jul 2012, 15:04
Coanda I like. But I like the best your thought that Bernoulli has it effect/cause, rather than cause, effect....

Air is 'transitional' for purposes of lift discussions, IMO. It can be thin, and weak, Stall, or solid, and abrasive, COLUMBIA. That makes it susceptible to those whose brain is well ordered, and given to Maths, and explaining things to simple people.

I am a Newton guy. Bernoulli was explained to me first by Hank, PA Captain. I revered this guy, being eleven, and worshipping all yhings flying.

I kept it to myself, but thought, Hank must be drinking again.

Owain Glyndwr
26th Jul 2012, 15:11
the faster the air is, the more accelerated it is
Why would that be?

Well, this one I must confess I am making an assumption. That assumption is that for a given set of conditions, the speed imparted to the air is a "given percentage". So the faster the air stream is, for that same "percentage" means that the air is imparted a higher speed. Something like: if airspeed is 100 kt, the air will be accelerated 10 kt (a 10%). If the airspeed is 200 kt, then the air will be accelerated 20 kt. Double acceleration, double force.

If I am not right, then there must be another reason why increasing speed increases lift, aside from more air being "processed" in a given instant. Otherwise speed wouldn't go squared.

Ah, I see what you are getting at. It was the use of " more accelerated" that fooled me - not thinking straight ;) As you say, if you double the airspeed then the mass flow rate over the wing doubles and the downwash velocity doubles (at fixed AoA). The wing lift then increases by mass flow rate* downwash velocity i.e. by a factor of 4 (speed squared).

Owain Glyndwr
26th Jul 2012, 15:18
Let's say the airplane is cruising at altitude at lower speeds and 100% of the thrust is from the turbojet. Nothing from the inlet or nozzle. The thrust is 100,000 lbs total and is consuming 100 gal/hr. Now, let's speed up to the normal cruise where the turbojet is producing 8% of the total thrust. Let's say the thrust is 200,000 lbs for normal cruise. 8% is 16,000 lbs and the corresponding fuel flow would be 16 gal/hr for that thrust. So, does this mean that only 16 gal/hr are leaving the fuel tanks when the airplane is in cruise?! We might as well get the numbers right. In one of the postings in the Concorde thread M2Dude gave some figures from RR. The cruise fuel flow was 9700 lb/hour/engine and the engine thrust was quoted as 8050 lb, but this may have included the secondary nozzle contribution as it was shown on the accompanying diagram and RR might well claim that thrust as part of the engine firms' contribution to powerplant thrust. At mid-cruise weight the required thrust per powerplant (intake+engine+nozzle) would have been about 10000 lb.

pattern_is_full
26th Jul 2012, 19:14
A thread on the theory of "lift" in which the past 27 posts are about engines and not wings...

Unless we're talking about Harriers, isn't that a bit much?

Lyman
26th Jul 2012, 19:38
Engines have wings too......

FlightPathOBN
26th Jul 2012, 20:28
If Bernoulli is correct, why do they put wing designs in a wave tank to test them!?!?! :}
Actually, using a wave tank is wrong anyways, because while you can compress air, but you cannot compress water.

Just think if the surface vessel designers, designing boat hulls and trim tabs, would meet an aircraft wing designer, I bet the bottom of the wings would look much different (and work much better)

Lets face it, Newton is correct, the aircraft planes through the air, just like a boat on the surface of the water, and the waveform off the bottom of each is very much the same.

While the wave from the surface vessel tends to roll outward, the aircraft wing wake turbulence would as well, if the component from the top of the wing did not cause a rollup at the intersection of the influences.

Boeing is testing the 777-B on Puget Sound right now..
http://meganparenteau.files.wordpress.com/2012/02/boat-with-wings.jpg?w=490

Microburst2002
27th Jul 2012, 06:27
Hi Owain

something like that, but as I say it is kind of an intuition. We can investigate this further.

The first part, the increased mass flow, I think it is clear: the faster the airspeed, the more mass of air is pushed, the more lift is generated. The other part, the increased acceleration due to increased airspeed, that is another matter, but seems logical: the stronger the push by the wing, the steeper the pressure gradients around the wing, the faster the air will flow as a result and the stronger the upwash and downwash effect.

that would explain speed squared

The truth is that the empirical approach is clearly the best.

Owain Glyndwr
27th Jul 2012, 08:33
Hi Microburst,

As you say, it is almost intuition! I think that in reality everyone will go for an explanation that suits their mind set. For me, a one-time aerodynamicist, I tend to think non-dimensionally, so I seek an explanation in terms of flow angles - AoA and downwash angle off the trailing edge.

Classical (inviscid) theoretical aerodynamics couldn't explain either lift or drag until a chap called Kutta introduced a hypothesis that the flow should leave the airfoil at the trailing edge. To make that happen he had to introduce a new variable that he called circulation, so that the flow over the airfoil was a combination of freestream plus a "circular" flow which added velocity on the upper surface and reduced it on the lower. Trouble was, to get such circulation in real life you have to accept a 'starting vortex' when the air first starts to flow over the airfoil. But vortices are viscous phenomena so to make the (inviscid) theory work you have to rely on the existence of viscosity :D

Anyway, in my non-dimensional world the angle of the flow coming off the TE is only a function of AoA and wing camber. If you grant that, then the downwash velocity is proportional to freestream velocity as is the mass flow.
From that lift proportional to velocity squared follows directly.

Microburst2002
27th Jul 2012, 09:05
yes

Darroll Stinton was also based on the circulation, I think. I wish I had that book here...

sometimes I wonder If I would be a happier person just using the theory of bernouilli as given in the books...

fizz57
27th Jul 2012, 09:06
I think people should give some thought as to what use the various theories should be put to.

At the "why do wings fly" level, Newton clearly rules,to use the terminology of this forum. A wing is a machine for producing downwash, period. The fact that an unstalled wing deflects more of the air passing over its upper surface than that over its lower is only a detail, however important.

Unfortunately to get numerically correct results Newton's Laws should be applied to every air molecule passing over the wing, which is impractical even with today's computers. So there are the macroscopic theories, concepts of pressure and energy, Navier-Stokes equations, circulation, etc. But these theories are intended for calculation, not for explaining why a wing flies.

Finally, Bernoulli (and Bernoulli's equations are ultimately derived from Newton's Laws) is probably the best way of explaining the vital need of keeping the airflow attached to the upper surface, without going into the detail of either the atomic or the calculation-oriented approaches. For those whose day-to-day business it is to ensure that the airflow does stay attached, it's an approach that has worked for over a century - although how well it actually works may be questionable.

Mr Optimistic
27th Jul 2012, 16:42
Just integrate the pressure over a control surface and have done with it

;). Potential flow plus circulation and diverting gaze from the odd singularity does it for me.

Microburst2002
27th Jul 2012, 17:59
fizz57, I agree with you

and I would add, for the indoctrination of those whose job is to keep the wing unstalled, what is the real problem of stall.

For most the stall is a problem of lift, when it is clearly not. Lift of a stalled wing is much greater that lift for level flight. the real problem is the greatly increased drag and the control problems. If an airplane had powerful enough engines and could control attitude even at very high angles of attack, flying while the wings were stalled would just be a problem of fuel. But airliners can't do that, and that is why they will sink if they stay stalled.

So yes, what all pilot should know, and it was supposed to know if from the first lesson, is that you have to unstall whe wing to eliminate the problem. Apparently many pilots did not have that very clear, as the whole industry has had to meet and embarrass the pilot community. Because it was embarrasing...

bookworm
28th Jul 2012, 15:50
At the "why do wings fly" level, Newton clearly rules,to use the terminology of this forum. A wing is a machine for producing downwash, period.

Usually? Maybe. "Period"? No. A wing in ground effect is not creating enough downwash to account for the lift.

I think you have to separate two questions.

1) How does one "explain" the empirical flow field that is observed around an aerofoil of a particular shape and at a particular AoA? (i.e. why does the air flow like that?)

2) How does one predict the lift produced by the aerofoil in that flow field?

"Newton" and "Bernoulli" as described here both help answer the second question. They don't help with the first. For that, one has to be satisfied with some waving of hands, or be comfortable with solving the equations associated with flow. I've never found much in between.

Mr Optimistic
29th Jul 2012, 07:57
Yes, indicative of the attention deficit that seems to apply once the general principles are believed understood and the crowd rush to other more sexy problems. Field problems are generally not that easy to interpret by cause and effect thinking. Easier to figure out steady state solution and not worry too much about the initial transient which was how the system got there . Think the unsatisfactory discussion on starting vortices begins here!
However for a structure in airflow midchord flow is forced down so momentum theory is happy and the interrupted flow causes a reduced pressure behind the wing which accelerates local air. So reduced pressure and higher speed above the wing and everybody is happy :O

Lyman
29th Jul 2012, 10:06
"Usually? Maybe. "Period"? No. A wing in ground effect is not creating enough downwash to account for the lift."

My thought on ground effect is that the wing functions in what is functionally denser air (ambient) due the piston effect of the wings. For a height of roughly one half the wing span, the air cannot escape the squeeze of the wing's downwash as quickly as in a fully 'open' region, the wing and ground proximity reduce the escape "area" between them; the local airmass is functionally, denser....

Pugilistic Animus
29th Jul 2012, 20:31
For the really masochistic or obsessed Abbott and Van Doenhoff's 'Theory of Wing Sections' answers everything...it is probably my favorite book in aerodynamics though...but it's a very rough read and you really have to KNOW math and physics {engineering level}otherwise it makes absolutely no sense, it's mainly a reference book:)

J_T great explanation...:ok:

VinRouge
3rd Aug 2012, 12:00
I use this website, it has excellent notes not only on aerodynamics but also on stability and control.

http://www.flightlab.net/Flightlab.net/Download_Course_Notes.html

I struggled with the theory of lift for 3 years at uni.
Couple of things I have in my head off hand which help me to understand:

Inviscid fluids cannot produce lift. This has been experimentally proven. There has been experiments with superfluid helium which prove this is true. Viscosity is essential for the production of circulation, which is a function of the Starting vortex (which you can see in the bath, push your flat hand through water at an angle of attack and you can see it). There is a theory (von Karmen I think) which states there has to be an equal and opposite vortex attached at the wing to maintain balance of forces.

2D wing sections do not produce lift. only 3D wing sections produce lift. 3 Dimensional effects are very important and form the heart of the "Bound Vortex" Theory of lift. This also explains downwash experienced by the fuselage aft of the wing and at the tailplane, changing pitching moment across the wing section and changing angle of attack experinced across the wing.

Without drag, there can be no lift. This comes back to both viscosity and also induced drag as a result of angle of attack of the wing to the freestream flow.

Lyman
3rd Aug 2012, 12:09
"You will never understand lift. Forget it. You haven’t got a chance."

And that is what makes me smile.... Whether from perversity or some other antisocial bent, I enjoy that man can not prove mastery over Nature. Some things are enjoyed in their mystery; it levels the field.

My motto regarding lift and it's tantalyzing resistance to explanation is,

Math hates RATE...... It's why pilots use their hands to explain things, and mathematicians use a pencil.

Microburst2002
3rd Aug 2012, 13:11
Abraham

Thnk you very much, reading your post has been delightful.

I always like to compare lift equation with friction force equation, when talking about lift theory.

Cambridge, Gottingen or MIT scientists could pursue and find an equation for friction which could be pages long, using all kinds of variables such as molecular mass and reach a point where physics would meet chemistry and even quantum theory, why not...

But then it is so much easier to make some experiments, tabulate, find relations of proportionality and find the constants for each material...

The equation obtained is so simple and beautiful, and so innocent because it does not challenge the mistery of creation, it only stares one of its laws. We could have an endless debate about "why is ther a friction force?" similar to "why airfoils lift".

That is why I love the lift Equation. It does not explain why air behaves the way it does when going past a wing. it accept it behaves that way. Then it describes how it affects the wing an what to expect if a few variables are known.

Lyman
3rd Aug 2012, 13:46
Yes The quantum. The transition. I really do not hate Math...much. I don't know if there will be a meeting of the maths with Mother Nature. But I say, soldier on!

I 'll be patient. In no other pursuit I know, is Faith a component of Science. It is why the old ones were called back to fly the 747. Gray hair and four gold, something to believe in. If he thinks it will fly, maybe it will.....

roulishollandais
5th Aug 2012, 20:08
@extricate:
if you prepare exam, flight licences, Atpl instructor qualification, etc. or if you want a well paid carrier in aerospace, stay on 1.
If you want to understand how birds fly, how unloading the wing works, how spacecrafts come back on earth, how butterfly flies, ...how nature allows lift goto 2!

1. If you want to get your exams, please, just write as it is written in YOUR books. Your teachers teach the lift as these crazy theories give them their salaries, to get famous, for some of them to get Nobel prices. And forget... (maybe you will finish like AF447 in the ocean, take a good insurance to help your family : all that is only statistics! "So many millions NM anf flights have not accident..." STOP.:uhoh: GOTO END

2. You want really fly like birds! Just look seriously the sky : can you really imagine the air is continuous, homogen, isotrope? NO NO NO. So forget ALL the continuous equations: Bernouilli, speed model, pressures equations like navier-stokes or Kutta,Lanchester :First learn Fractals,(it is difficult),dynamic systems (sampled fractals...), and plastic rupture! It is a nice story. Good kuck. And never forget. STOP.:ok: GOTOEND.

END.

Volume
6th Aug 2012, 09:42
For most the stall is a problem of lift, when it is clearly not. Lift of a stalled wing is much greater that lift for level flight. the real problem is the greatly increased drag and the control problems. I do fully agree with the first sentence, but at the same time do fully disagree with the second.
The problem of stall is not lift, it is the slope of the lift-over-AoA Curve! The Airplane can fly at any Cl, given the right speed is applied. So a drop in lift can be compensated by more speed, which can be easily demonstrated in Gliders which have two Cl max peaks in the lift-over-AoA Curve. In full stall some of them fly fully stable, but with a higher speed than the lowest possible one. (You can easily demostrate that when flying in formation)
The equilibrum of forces perpendicular to the flight path (or the airflow) is only stable, if there is a positive slope of the lift-over-AoA Curve. Lift automatically equals weight as long as the slope is positive, mother nature controls this for us. Any lack in lift results in increase of (downward) vertical speed, which results in AoA increase which restores lift. If however the slope of the lift-over-AoA Curve is negative, then lack in lift results in increase of (downward) vertical speed, which results in AoA increase which further reduces lift, hence the airplane switches from the modern rules of aerodynamics to the ancient law of gravity.

Nobody worries about the theory of gravity, neither should we worry about where lift comes from. As long as we know how to loose it, and avoid that situation, we are fine.

Microburst2002
6th Aug 2012, 14:37
Volume

I don't agree with your disagreement.

Lift coefficient at very high angles of attack is very high, and in the stall, even well inside it, it is very high too, compared with typical cruise angle of attack.

Of course, if you keep pulling and pulling and your aoa increases a lot, then even CL will be low. We agree in that. But even in that case, if you have enough power and means to maintain a given attitude steadily, you can fly the airplane. However this is extremely difficult, specially the control part.

So if pilots were indoctrinated in this, maybe using modern fighter jets as an example, they could be convinced than pushing the stick is the right thing to do whenever they face a stall or impending stall situation.

In my flight school, long time ago, in the piper arrow we practiced a manoeuvre called something like "characteristic stall", which was a progressive stalling of the airplane in landing configuration. The exercise was successful if after the stall warning you recovered the stall... without losing more than 20 ft. Many times this exercise was carried out at merely 500 ft, over the runway!

I had lots of arguments for that with instructors, but I only had about 120 hours. Now I know I was right. The whole industry met only to clarify my point.

Negative training is intensive and extensive and it is everywhere. One day they should address it seriously, it deserves more attention.

john_tullamarine
6th Aug 2012, 21:47
Many times this exercise was carried out at merely 500 ft, over the runway!

How quaint. How does one spell "silly" again ?

While specific techniques may vary between Types, the general rule with stalling is that alpha has to be reduced somehow in order to escape ..

Pugilistic Animus
8th Aug 2012, 12:48
I'm really loving this discussion...I wish I had time to really join in the way I'd like to...:8

but I don't have the time really so if I make make two brief points
1...the best mind in aerodynamics who ever existed was SzQllQskizlaki Von Karman Todor...better known as Theodore Von Karman:ok::ok::ok:

2...a designer picks whatever equations s/he need to meet the design the theory of differential equations goes on seemingly forever...:)

One of the main reasons I don't have enough time to post what I'd really like to say sometimes is that my laptop (girlfriend) keeps hogging my desktop...confused?:confused:
well here's what I mean

http://press.princeton.edu/images/k7999.gif

:}:}:}:ouch:











ps I think she's doing a vg diagram;)

Pugilistic Animus
8th Aug 2012, 13:08
http://ts1.mm.bing.net/th?id=I4920572877669752&pid=1.7&w=217&h=155&c=7&rs=1

what he has written on the board is what exactly i've been rambling about all along...:)

roulishollandais
18th Aug 2012, 18:56
Hi Pugilistic animus !

Karman of course ! and Libchaber, Lorenz, Feigenbaum, and so many great great great physicians, mathematicians....

RIP taboos, and Boltzmann, Navier or Stokes unsolvable equations.

Welcome to all the real BIRDS who know all that from the nest... :) I am jaleous !

The AF447 discussions shows that it is essential to open the door to future.... Really !

Pugilistic Animus
19th Aug 2012, 10:41
roulishollandais it's not really that there's 'no solution' per se, it's just that there's no closed solution to the NS formulations...a complete examination of the equations of motion and thermodynamics, with repect to fluids yields two very important non-dimensional quantities...Mach Number and Reynold's number--- that is exactly where ALL of our problems arise...:\:\:\

Aerodynamics, is for the most part an experimental art, and it is an art, as so many technical field's really are...

Mad(flt)Scientist has a habit of revealing some of the secrets of design on pprune...for serious students of the field I believe his posts are definitely worth a read...;)

of course we must all remember two very important thing to progress with a design... for better or worse, new ideas are always dangerous and that nature laugh at complex mathematics...laughs!!!


:):):)

John Farley
19th Aug 2012, 14:12
Aerodynamics, is for the most part an experimental art, and it is an art, as so many technical field's really are...


Isn't that the truth.

It always makes me smile when people get adamant about Newton in discussing where lift comes from. They seem to totally ignore the fact that wind tunnels have been around for quite a while and in those it is easy to measure the actual pressure at the wing surface on both the top and the underside.

I was brought up to believe that to be correct a theory had to fit in with the facts. Hey ho.

mike-wsm
19th Aug 2012, 14:26
Wind tunnels are very useful for making measurements but it is important to acknowledge that they are physically bounded so that airflow is always horizontal at the upper and lower boundaries. In free air the flow differs. Traditional aerodynamic theory is based on dimensional analysis and simple algebra, but with the power of modern computer modelling it would be possible to run large-scale elemental simulations based entirely on Newtonian mechanics and thermodynamic behaviour of each element.

Pugilistic Animus
19th Aug 2012, 17:36
Ah the Professor and Mr. Farley, great company indeed!

:):):)

Lyman
19th Aug 2012, 19:47
Humbling, eh, PA? Only a few blokes could make me doubt Newton...

roulishollandais
19th Aug 2012, 19:54
complex mathematics... "Math pures" complexity start as soon at ... zero ! 1 is not simpler, etc. . Heappily we have "mathematiques appliquées" and "calculs astronomiques" to do it simplier ! And Libchaber two little turbulence are no more complex at all !


roulishollandais it's not really that there's 'no solution' per se, it's just that there's no closed solution to the NS formulations...a complete examination of the equations of motion and thermodynamics, with repect to fluids yields two very important non-dimensional quantities...Mach Number and Reynold's number--- that is exactly where ALL of our problems arise...http://images.ibsrv.net/ibsrv/res/src:www.pprune.org/get/images/smilies/wibble.gifhttp://images.ibsrv.net/ibsrv/res/src:www.pprune.org/get/images/smilies/wibble.gifhttp://images.ibsrv.net/ibsrv/res/src:www.pprune.org/get/images/smilies/wibble.gif

I did it a little short whith NS ! as I am a little lazy with words (in my "fractured English" as Organfreak said it to me... ) : "no analytical solutions" of course (I knew you knew!). And computer solutions don't get much to see before us (distance and time). And what with cavitation..

new ideas are always dangerous
Marcel Dassault had that art to be very prudent with "new ideas".

Wind tunnels are very useful for making measurements but it is important to acknowledge that they are physically bounded so that airflow is always horizontal at the upper and lower boundaries. In free air the flow differs Henri GIRAUD was a geant data base of airflow in free air when he died in nov.1999. I asked to the INRIA of Grenoble if they could work with him, it would have been yes, but as I asked to "Monsieur GIRAUD" he said me "No, it is too late". The civil aviation denied Henri GIRAUD importance during his whole life.


Shall visit Mado with your recommandation !

Volume
20th Aug 2012, 07:43
Windtunnels just like flow dynamic computer codes are alwys just a model of reality, and none gives exact results when compared to free flight measurement. Many phenomena have been discovered in free flight and could not be confirmed in the wind tunnel or by calculation. It is hard to produce a real free air stream in a wind tunnel, there always is some turbulence created by the fan or by the walls of the air intake, there is always some slight speed gradient close to the walls, there is always some restriction in the air flow if you add walls. Vice versa any computer code can just predict what has been part of the model from where formulas have been derived.

A recent example is that a turbulent boundary layer has been found to become laminar again under certain conditions, while mathematics tells you that this is impossible, and wind tunnels can not reproduce this.

Any theory is only good until a better one comes along. Any wind tunnel is only as good as the criteria it was designed for. Any measurement method (pressure by piezo sensors, speed by LDA, drag by wake analysis...) only works if you exactly know what phenomena to measure. There is not a lot of static flow in a steady flight, there is a lot of small amplitude high frequency variations in the parameters required to fully describe certain phenomena.

It looks like we understand aerodynamics for some hundred years, but in fact we are still learning something new every year.

roulishollandais
21st Aug 2012, 20:39
agreed ;)
Let us try to get better teachers to young pilots :)

John Farley
22nd Aug 2012, 12:06
Let us try to get better teachers to young pilots


A good point, however what they teach needs careful consideration.

Being up in the air some way above the ground with an aircraft strapped to your back is essentially a practical problem not a theoretical one.

In my view pilots need to be taught the factors that affect lift not the theory of lift. They may well be interested in the theory – and why not for goodness sake – but not at the expense of thoroughly understanding the factors that determine the lift available when they are flying.

The lift equation contains all the factors but only one term in that equation (the lift coefficient) is available for the pilot to vary in order to adjust the lift during the normal course of flying. It is the lift coefficient that is controlled by pulling and pushing on the stick (or wheel) as that varies the Angle of Attack on which the value of the lift coefficient depends.

Only Angle of Attack not speed makes a wing stall. Believing in that and living your life by it is vital to junior (and senior) pilots . The theory of how lift happens will not help you when turning finals. Only a proper appreciation of how to control lift will keep you alive.

roulishollandais
23rd Aug 2012, 20:40
The pilots of AF447 seemed to be ignorant of AoA.
Airbus considered that showing the AoA on the display was not useful.
Certification administrations accepted such equipped aircraft to be used for passengers public transport, not matter of AoA.
Air France buyed that aircraft without AoA display.
Pilot unions (SNPL SNPNAC SPAF UNAC, etc.) did never any strike to get AoA.
In other Countries it was the same.
ICAO did no global recommandation.
AESA accepted that mix of AoA ignorance.
WHY?
Many pilots knew enough physik before to get pilot (math sup or mor fo someone.
BUT : At the first page of any aerodynamics book for airline pilot you see the figure of a wing with airstreams thiner when passing over the wing : Well! And then it is written that these airstreams are independant (Bolzmann theory of gas). Question:How does the third airstream knows the wing is there to get thinner?
The pilot trainee decides then to forget definitively the logic and the physic laws!!! And in flying lessons it is said that good pilots are "stupid"...
NO NEED OF AOA!
Some aerobatics flights would SHOW him he is wrong. But it is the time for "NEW METHOD" (Thank you Monsieur DEBIESSE, who initiated that method in the French DGAC)
NO NEED OF AOA SENSORS!
Only a good theoric teaching can bring back confidence to physic reality.

FlightPathOBN
23rd Aug 2012, 21:04
mount a bubble in the cockpit...its as easy as that.

Old school AoA angle and inertial speed..
http://operationsbasednavigation.com/wordpress/wp-content/uploads/2011/04/INSoldschool-e1345756120229.jpg

on a side note, if the pilot has an iPad, use the level bubble app.

there are many concepts in the works that use these types of concepts...

Pugilistic Animus
23rd Aug 2012, 21:14
Just to make a quick point...regarding Rn Mn

we simply use Rn as a rough guide depending on what types of forces are dominant-to predict flow separation point and when laminar becomes turbulent---in reality it's not so simple as there are usually mixed flows...plus the interaction with shockwaves etc

At slow speeds and low altitudes and 'conservative' airfoils rather simplistic treaments can be used...as one goes higher and faster it gets progressively harder; but they try to get it as good as possible on paper---and these complex treatments are for designers-not pilots; the wind tunnel provides a better picture and is still cheaper than lots of full scale models provided you do your tests at representative Rn and Mn [or EAS] ---but the aforementioned limitations---described by mike-wsm and Volume---also prevent a full appreciation of the flight characteristics---only through flight testing can we be sure---but even at the most esoteric level---Newton still rules---i.e wing pushes air down---air pushes wing up---that never changes---the mathematical mess is for designers---John Farley's word once again ring true...Brian Abraham's synopsis of the various formulations is very interesting just hope this post puts it into some more perspective...also there are many different treatments with different limiting conditions different acuracies/precision depending on exactly what you're doing...Von Karman corrected most of Prandtl's formulations...he was probably the most exacting one...;)

roulishollandais
your assesment is correct 'pilots are not supposed to be too smart'..:}:ouch:
and don't worry about your typing I'm a native English/Spanish speaker and I can guarantee that at times mine is much worse...:uhoh:

Il y'a etait depuis tres longue temps lorsque j'ai parlait Francais aussi ---J'ai oublie comment tout de mon Francais...I think I said that right...sorry for the missing accent marks too lazy... :)

I forgot most of my Italian too...:uhoh::uhoh::uhoh:


:):):)

john_tullamarine
24th Aug 2012, 00:11
Old school AoA angle and inertial speed..

Could be just my dotage and old timer's disease taking hold again .. but, how does that work ?

I can follow the argument for body angle (rather than AoA) under non-accelerated steady state flight conditions but no more than that.

PJ2
24th Aug 2012, 00:27
John...I was wondering the same thing!

Pugilistic Animus
24th Aug 2012, 00:45
Now that I think of it I get AoA part but the inertial speed???

why can't one just use AoA in isolation? ... unless are you discussing the lift slope curve dCl/alpha you could get the TAS in order to compute Cl and perhaps through that get a lift slope curve, but I dont think that's a usual method, just guessing though... :)

john_tullamarine
24th Aug 2012, 02:13
.. an inclinometer knows naught about AoA, the latter involving the windy bits as well as where the wing might be pointed ? and not much about anything if there be other than gravitational forces involved.

In the same category as the old furphy about using a pencil suspended on a bit of string to let the pilot fly in IMC without an AH etc.

Or am I just getting too old for all this heavy stuff ?

Pugilistic Animus
24th Aug 2012, 02:55
Oh J_T I was on my phone not my laptop, couldn't see clearly...I thought that was an AoA indicator...now I see; you can't get any information from the above set -up...but you, as you know, can get the curve from AoA and TAS data...:)

john_tullamarine
24th Aug 2012, 03:06
Pass, brother.

Brian Abraham
24th Aug 2012, 03:40
why can't one just use AoA in isolationYou can. Naval aviators use it as the prime source on a carrier approach.
In the same category as the old furphy about using a pencil suspended on a bit of string to let the pilot fly in IMC without an AHYou not telling me you can't JT? :E A certain test pilot from over the water not far away writes in a book that that was his back up on a round the world venture.

john_tullamarine
24th Aug 2012, 03:43
But I'm only an average jock with low level test work under my belt. Not up the the standard of a real TP.

I have no problem with the pencil trick .. it's just that I get confused when upside down during an I/F loop and it's still telling me I'm straight and level ... just one of my many shortcomings, I guess .. all happily married men are well aware of their many shortcomings ..

that was his back up on a round the world venture.

OK if it's a TSO'd pencil on a string ..


We still have to catch up for that beer at Jack's ...

bubbers44
24th Aug 2012, 04:04
I had no ground school, just bought books until my instrument rating. Worked great for me. I even got my instructors rating reading a book. I went to take the test because I wasn't sure how to study for the test because of all the philosophy of teaching BS. Well, I passed so never learned it. Taught my self aerobatics out of a book, then taught it for Art Scholl, one of the best, with his school. Books are great if you can do it without someone to help you.

My first job when I got my commercial license was crop dusting with zero training. Loved it. After soloing with 5 hrs total I was doing solo spins and finally a loop solo before 10 hrs. It was fun learning that way. With 5 hrs back then you could fly cross country anywhere you wanted to go so would go out and land in the desert on the sand just for fun. I had a AAA map for nav, no aviation charts and did just fine. That 15 degrees variation didn't bother me a bit because at that time I didn't know what it was. Well guess it was in one of the books.

Ended up flying with the largest airline in the world at the time flying a B727. Ended up in the B757 and B767. Funny how things can end up mainly reading books, isn't it.

john_tullamarine
24th Aug 2012, 04:19
Naught wrong with reading good books, my friend.

I am in two minds about the internet, though. On the one hand, it opens up a mass of links to all sorts of good stuff .. but, at the same time, it isn't a patch on lounging back in the rocker and flicking through a good book.

bubbers44
24th Aug 2012, 04:19
I guess what I am saying is don't blame who trained you if you don't train yourself and use your own initiative. We should all take it upon ourselves to be as competent as possible and not throw that responsibility on your company. Take charge yourself.

john_tullamarine
24th Aug 2012, 04:36
We are in heated agreement.

bubbers44
24th Aug 2012, 04:43
JT, my first instrument class was at a junior college and was a total waste of time, had to go back to the books. I know things are different now but this was in the 60's. Usually the ground school teacher reads the book and repeats it soon after the best he can.

My roomate when I was flying a Jetstar was teaching ground school in a high school and I told him he was teaching it wrong. He passed the private pilot written exam but didn't understand what he was teaching. His students would have been much better off reading the manual than listening to him.

I went to Flight Safety in Teterbero in the 70's and he was explaining how to use reversers on a Falcon 20. He said pull the TR levers back then advance the throttles. My Mexican students were going into the sim that night and I said just pull the thrust levers back, don't advance the throttles. They both said no, he said push the thrust levers forward. I just said see you in the morning. Of course the next morning I asked did you push the thrust lever up and they sheepishly said no.

Sometimes reading the manual is the best thing if your instructor is weak.

bubbers44
24th Aug 2012, 04:56
JT, I agree. Us old timers grew up in a different time and it doesn't work this way now. Now we have a lot of rules that I am happy 9 yrs ago I could leave at 60. I loved the profession but it wouldn't be as much fun now with all the new stuff. I spent my last years flying into Tegucigalpa, Honduras just because the only way we could land was to break all the present rules like 1200 fpm to flair or you couldn't land. We had to fly beside a hill and were not wings level in a B757 until 100 ft, loved it plus the 70 ft cliff at the end of the 5400 ft usable runway made you do it right every time. That is the only part I miss.

john_tullamarine
24th Aug 2012, 05:06
Then, at some stage, we should have a convivial ale and reminisce on such things.

Certainly, having done a lot of class room teaching over the years ranging from tertiary aviation/engineering through secondary math/science and on to pilot training, unless the instructor is many levels above the student in technical competence, it is a waste of time. No point in the blind leading the blind.

Pugilistic Animus
24th Aug 2012, 07:23
Mr. Abraham it was a semi rhetorical question re: AoA...;)

I taught myself how to fly...of course legally you need an instructor but for the most part it was a game of Simon says...i.e
CFI says do this...ok
CFI says, do this...ok
do this....no, no,no, need to use the checklist first

Mostly me and my instructors talked about girls, life, plane crashes, etc... never the task at hand...I almost miss those days...:)

roulishollandais
27th Aug 2012, 16:55
Math hates RATE...... It's why pilots use their hands to explain things, and mathematicians use a pencil



All the flying birds fly without rudder nor fin.
Fortunately, birds do not have rigid tails. Being supremely skilled at "hand flying" they do not appear to be in need of many protections. http://images.ibsrv.net/ibsrv/res/src:www.pprune.org/get/images/smilies/wink2.gif

http://t2.gstatic.com/images?q=tbn:ANd9GcT6YTBlHkR7vcT5QkN7waYM-1y946lkPM5EqzBs_IKel8ZwzMjrhttp://t1.gstatic.com/images?q=tbn:ANd9GcTYqBIxDjt5Lt5Vz1NFld-BF91bRBSajs-ctuk4WBNaZYT5RlFNFA



Thank you md80fanatic for these two beautiful examples of no rudder nor fin :)


@ Lyman
Just look the shape of the wings in math language.
They have fractal flight law and not continuous function like NS :p

ruddman
28th Aug 2012, 10:51
Lot's of credit to man. No mention to the one who understands lift better then all of them put together.Two of his designs pictured above.



Credit where credit due...:D

Pugilistic Animus
29th Aug 2012, 03:13
Ah yes, indeed, flight of animals-immensely complex--it's quite a shame we can't go to the birds to obtain the answers really...:}

Not sure about fractals and such; they use Newton too...they just know what the hell they're doing unlike us...:ooh::ouch:

:}:}:}

roulishollandais
29th Aug 2012, 19:30
Less complex that Navier Stokes equations!
We would like to ask the drop of water in the river flowing toward the bridge how they recon their acceleration, speed, position, spin, deformation, ASO!
:*
Newton? Of course! Like for galaxies from HENON and Laurent NOTTALE. The latter uses fractals to join Relativity theory and Theorie quantique (Paris -Meudon Observatory CNR'S Dr of Research) .:O

Pugilistic Animus
29th Aug 2012, 22:31
roulishollandais
I didn't say you were wrong or anything just that I am unfamiliar with fractals as a part of conventional aerodynamics, as I've never seen such a treatment before within the subject...I have some idea about fractal geometry but I admit I'm having some trouble with your line of reasoning...but I guess there's always more than one way to peel a potato...

Bon soir

:)

roulishollandais
30th Aug 2012, 17:32
http://www.newscientist.com/blogs/culturelab/2012/08/23/51%2BV7-46DdL._SL500_AA300_.jpg
Hi Pugillistic Animus,

I am glad to tell about fractals also known as Theory of chaos.

This excellent and cheap book (1987) tells many things about fractals and mekanik of fluids.

Chaos: Making a new science (http://www.vintage-books.co.uk/books/0749386061/james-gleick/chaos/) by James Gleick (1987)
The first popular science book to tackle the emerging field of chaos theory, journalist James Gleick’s Chaos earned the author a Pulitzer prize. Not only did the book bring this complex physics to the public, says our features editor Ben Crystall, “It helped kick-start the subject into a host of other fields”.

This book exists also in French pocket version and new edition is from 2008. (editions Flammarion, 10€ !)

I didn't say you were wrong or anything just that I am unfamiliar with fractals as a part of conventional aerodynamics, as I've never seen such a treatment before within the subject...

Why is the conventional aerodynamics still ignorant of fractals ? You know, for a long time a divorce existed between mathematicians and physicians. Physicians wrote some equations, writing integrations of not continuous functions for example, doing mathematicians angry. But math had other goals, like topology, math for computers, and was late to help physicians. Something changed really in the middle of the 20.century with "fractals".

With fractal geometry and theory, sundely physicians were behind, they were struggling with automation and theory of systems, and mathematicians had advantage. In reality we need to work together... but it is not enough to say that !

Things changed when fractals brought new solutions for non-linear dynamic system, new thinking.

As always in science it needs still 15 years or more to update technology.

Discussing C* about AF447, I discovered that derivating don't go higher than 2. (position, speed, acceleration). Why not tendance of acceleration, for instance ? Every thing needs TIME !


I have seen you are very active in the climate change debate. The meteorologist Edward Lorentz has been a great pionnier of climate prevision and discovering the meteorologic part of fractals...

Read Gleick ! You will get good surprises !

Pugilistic Animus
30th Aug 2012, 17:38
I certainly will check it out mon ami---I just hope it's written so that my little brain can understand...engineering and science math is very easy--- just tedious that 'math-people's math is almost incomprehensible to me...i'm no mathematician that's for sure

roulishollandais
30th Aug 2012, 19:50
Hi Pugillistic Animus,
You will see birds, rivers, hurricanes, FCS, etc. otherwise after reading James Gleick about fractals ... A good index will help you to sort different matters after a first cross-reading (just a suggestion). Math is often showed as interesting at abstract objects, Nobody knows what an abstract object is ! We have only senses to discover new objects. In our brain we can only play with them !:ok:

Some teacher will tell about "linear combination" :Eanother about "linear shape" :) for the same thing : you see, smell, taste, listen, and touch the latter, the first stays a mystery... Go with the good teacher !

Faites de beaux rêves !

Pugilistic Animus
30th Aug 2012, 20:45
Je vous voudraise la meme chose

merci beaucoup...:)

Yankee Whisky
30th Aug 2012, 21:28
The picture shown in the FAA explanation makes no sense to me.

To look at the rotational direction, it shows pressure increasing at the bottom and decreasing at the top, hence a resulting up force.

Should this not be the other way around because, the way I understand it,
the cylinder part rotating into the wind creates an increase of the relative airflow speed (pressure drop) over the surface (in the illustration; at the bottom) and the top shows relative airflow slowed down (pressure increase) accross the surface.

Are there effects causing my understanding not to be true ?

Just curious, because aerodynamics have served me well in my 60 years of flying.:confused:

chris weston
30th Aug 2012, 21:56
121 Sir

Flying is a matter of surfing on a mixture of gases, most of the rest is simply managing the drag using tedious mathematics.

If you rotate a rigid cylinder as suggested there will be more collisions with the gas phase per unit time with that structure than if it were static.

Hence more lift.

Pugilistic Animus
30th Aug 2012, 21:59
Nope the model is correct note the acceleration of the flow in the 'torus' model

The relative wind [flow] is accelerated at the top---which 'pulls' the bottom flow into greater density hence higher pressure--- one wants a pressure increase on the bottom and decrease at the top the FAA publications are all very good

airplane flying handbook,glider flying handbook, aircraft W&B handbook instrument flying handbook, helicopter flying handbook, pilot's handbook/encyclopedia of aeronautical knowledge, seaplanes, ski planes and float and ski equipped helipcopter flying hand book...and although this is biased to FAA rules the instrument procedures handbook and they are very cheap too... those are what I use for ground school with my flock...very accurate!!!
there is some some blatant plagiarizing from Hurt and Davies' but that's ok the government can steal...look at Taxes...:}

:)

Pugilistic Animus
30th Aug 2012, 22:09
two things I meant Rotorcraft flying handbook they cover gyroplanes too

and I realize I'm not being 100% perfect in my explanation like 98%...trust me the real story is quite involed no need for that here... I'm trying to relax too...:)

Edit Chris Weston that's coolway to see it...:):):)

Pugilistic Animus
30th Aug 2012, 22:21
I just needed one more post...cuz I'm a little bit superstitious...:ouch:

this hotty has the theory all worked out...:cool:
Svetlana Kapanina Aerobatic Pilot. - YouTube
boy would I love to do rolling circles and humptybumps with her...:}

Another lovely lady who I love
http://www.youtube.com/watch?v=wx_ui2qWgqI

jcbmack
31st Aug 2012, 18:47
But this is all so very interesting--will keep reading and watching.

Pugilistic Animus
31st Aug 2012, 19:03
Jcb do not be disheartened in some ways aerodynamics is out of everyones's league even Mr. Boeing and Mrs. AirBus

Pugilistic Animus
31st Aug 2012, 22:24
I'd just like too add a few more cheaps books to the ones I listed above

FAA aviation weather..for folks flying in the US ---Aviation weather services

Buck's Weather Flying... Taylor's Instrument Flying..Langwiesche's Stick and Rudder...Webb's Fly the Wing...Mike Goulian's and Geza Sourvizoy's basic and advanced aerobatics... Mallinson and Wollard's Handbook of Glider Aerobatics...good stuff for airplane pilots too

For those who aren't engineers but are interested in aerodynamics H.H Hurt's Aerodynamics for Naval Aviators

For those going to jets of course DP Davies Handling the Big Jets listed earlier
in addition to the book I

if you bought them all they's probably set you back about 300$ or so

I wouldn't recomnend pilots read any of my engineering aerodynamics books though...:\:\:\:eek::eek::eek:...seriously!!!


:):):)

roulishollandais
6th Sep 2012, 19:34
Hi John Tullamarine, Pugillistic Animus, Bubbers44, other books lovers,

Perhaps could PPRuNe's "Theory on lift "thread have a list of useful web links (like AF447 list, John ! :ok:) and useful good books ? :)

peter kent
13th Oct 2012, 00:30
Hi Italia,
Don't know if you are still receptive to getting to grips with the J58 as it's been a while since your June post. I don't believe anyone followed up on your points.
Incidentally, Ryman had already correctly killed the TR.
Some of the information in the sources you are referencing is wrong, including classifying the J58 as a TR, unfortunately. One source will reference another source and misunderstandings/errors get perpetuated despite their dedicated authors.

TR was probably coined by someone because it sounded more exotic than bypass bleed engine (which is what P&W called it). They had to fit Mach3's worth of impressive ram into the name somehow.

What follows only needs to look at the engine airflow, not the very significant airflow which goes around the engine and causes so much interest in the intake and exhaust thrust contributions.

First we'll see how similar the airflow is to that in a familiar Mach2-type military turbofan or bypass engine. Then we'll see the reason why it could never have been a TR.

Going one step at a time thro each engine, an F100 at Mach2 say, and the J58 at M3.

F100: all the air destined for the engine goes thro its first compressor, also called the fan.
J58: all the air destined for the engine goes thro the 1st 4 stages of its only compressor

F100: some of the fan exit air goes down the engine bypass duct, the rest goes thro its 2nd compressor ( and combustor and turbines)
J58: some of the 4th stage exit air goes down the 6 external engine bypass or bleed tubes, the rest goes thro the remaining 5 compressor stages (and combustor and turbine)

F100: the engine bypass air mixes with the turbine exit air upstream of the afterburner fuel manifolds
J58: the engine bypass air mixes with the turbine exit air U/S of the A/B fuel manifolds

These air paths are for all intents and purposes identical. If you wanted to classify the two engines you could even put them in the same drawer but it wouldn't have TR on the label. After the above you may wonder what was so special about the J58. Apart from the huge challenges overcome with materials, etc. it was different to an F100 type at lower speeds though because the bleeds were closed. So it had a bypass ratio varying from zero at low speeds to some higher value at cruise. This variable bleed made it unique. An F100 BPR just varies a bit about some nominal value.

The TR could have been, on paper at least, but it wasn't as the following shows (ref 'The Engines of Pratt&Whitney' by Jack Connors).
Early versions of the J58 were a plain afterburning turbojet (shown on many web photos).P & W converted it for M3 cruise by adding 6 bypass bleed ducts from the middle of the compressor to the turbine exit (shown on many web photos).
Alternatively, it could have been modified with blocker doors to close off the compressor inlet and a big annular passage around the engine for the ram air which would discharge into a common afterburner. So here, finally, is your TR, but it never happened because it would have been more mechanically complex and heavier. The bleed tubes were a much more elegant solution by virtue of their simplicity. They did the job.
The crucial insight here is you can't turn a turbojet into a TR just by adding a few bypass tubes.

Not wishing to belabor a point, an authoritative source on ramjet configurations (avail on the web) is 'A century of Ramjet Propulsion Technology Evolution' by Ronald S. Fry.
And for the jet engine an equal mine of essentially first-hand, and therefore accurate, information (avail on the web) is 'Gas Turbine Technology Evolution' by Bernard L. Koff.

I hope you have found the above of some value.

italia458
13th Oct 2012, 16:54
Thanks, Peter! That was a nice clarification.

In Figure 5 (http://www1.nasa.gov/centers/dryden/pdf/88507main_H-2179.pdf) there are 3 large diameter tubes running along the mid-portion of the engine - those are the bypass tubes, correct?

McBruce
13th Oct 2012, 21:29
What's others take on the whole theory for a symmetrical aerofoil in level flight? Could never get a good answer for this one out of the books!

A Squared
13th Oct 2012, 22:38
A symmetrical airfoil in level flight has a positive (greater than zero) angle of attack. is: The airfoil may be symmetrical, but it coes not encounter the airflow symmetrically. The airflow patterns around a symmetrical airfoil with a positive angle of attack are very similar to a cambered airfoil, with a greater amount of air flowing over the "top" of the air foil, at a higher velocity than the air flowing underneath.

A symmetrical airfoil at a zero angle of attack will produce no lift, but that ain't the same as level flight.

Could never get a good answer for this one out of the books!

Haven't been reading the right books then. Off the top of my head, I think that "Aerodynamics for Naval Aviators" gives a clear explanation of this, although I don't have a copy in front of me at the moment. Many other books on aerodynamics also explain this, including diagrams and photos from wind tunnels using smoke streamers to show airflow.


There's nothing mysterious about this at all.

Lyman
14th Oct 2012, 00:53
Even an asymmetrical profile does nothing without AOA. At zero AoA its lifting force is downward.... The upper surface has negative AoA due its fatter presentation vice lower, flatter plate.

It is a trick, parlayed into fame by the Swiss plumber.....

See, by the time the flow separates evenly, the aft portion of the flat plate on the bottom has gone Newton.....

A Squared
14th Oct 2012, 01:47
Even an asymmetrical profile does nothing without AOA. At zero AoA its lifting force is downward.... The upper surface has negative AoA due its fatter presentation vice lower, flatter plate.

It is a trick, parlayed into fame by the Swiss plumber.....

See, by the time the flow separates evenly, the aft portion of the flat plate on the bottom has gone Newton.....


Absolutely, 100 percent, purely wrong. Sorry to be blunt, but this is completely untrue. A typical cambered airfoil does produce considerable lift at zero angle of attack.

Here is a wikipedia article about angle of attack (http://en.wikipedia.org/wiki/Angle_of_attack) It includes a graph of coefficient of lift for a typical airfoil. You will see that the coefficient of lift at zero AoA is non-zero and positive.

If you're not inclined to believe Wikipedia, here's a page from NASA (http://quest.arc.nasa.gov/projects/aero/centennial/wrightflyeranswers.html) on the wright flyer, which includes another graph of lift coefficient vs AoA. It also shows non-zero, positive coefficient of lift at zero AoA. Additionally it contains a table of lift force, and a graph of same. Both show lift force at zero AoA to be non-zero and positive.

Here's another page (http://www.aerospaceweb.org/question/aerodynamics/q0136.shtml) showing a a graph of, among other things, the lift coefficient data for the airfoil on a Cessna 172. Coefficient of lift at zero AoA??? non-zero and positive.

Here's a page from the website of the US centennial of flight. (http://www.centennialofflight.gov/essay/Theories_of_Flight/Two_dimensional_coef/TH14G3.htm)

Care to make any bets as to what's on that page?

If you guessed that it contains a graph of coefficient of lift vs AoA, showing Coefficient of lift to be non-zero and positive at zero AoA, you'd be correct.


Obviously, if your understanding of the facts is this flawed, any conclusions you have drawn from the is inevitably equally flawed.

Lyman
14th Oct 2012, 02:09
Howdy.

The wing is not connected at leading edges, nor at the trailing edges.

It is hollow, and typically cambered, fat section forward. In the wind tunnel, air starts to flow over the wing, at zero angle of attack. The upper surface wants to move, which portion of the upper surface leaves first, trailing edge, or leading edge?

IOW, where is the center of lift on the upper surface, chordwise? Behind the equidistant point LE/TE? And how does this orient the chordwise axis to the longitudinal centerline? Negative?

You have no tail surface, only wing. How will you fly? Worse still, how will you increase the decreasing AoA?

italia458
14th Oct 2012, 04:43
I have to agree with A Squared. Lyman, you're talking about a phenomenon that is separate and more involved than what was being discussed. The simple, and correct, answer to what was being discussed is that an asymmetric wing will produce positive lift at zero AoA. How the wing is held at zero AoA is not relevant.

Just to be clear - the discussion so far seems to be discussing the geometric AoA, which is in reference to the chord line. The zero lift AoA is the geometric AoA required for a lift coefficient of zero. Absolute AoA is the geometric AoA minus the zero lift AoA - any airfoil, symmetrical or not, will produce zero lift at zero absolute AoA.

The zero lift AoA of an asymmetric airfoil will be negative. The zero lift AoA of an asymmetric airfoil will be zero.

Angle of Attack Awareness and Angle of Attack Management [Ch. 2 of See How It Flies] (http://www.av8n.com/how/htm/aoa.html#sec-raoa-aaoa)

http://faculty.washington.edu/lum/aa311/lecture_notes/lecture27.pdf

Lyman
14th Oct 2012, 05:30
The lift does nothing useful, it requires long methodical experimentation, and months and years of frustration, snd many dead airmen.

Ahhh....."wiki". So many sharp as a tack partisans, so much smarter than the average bear...

Those arrows, are they on your airplane? There are arrows, right? I must look sometime...

:ok:

Italia: "The zero lift AoA of an asymmetric airfoil will be negative. The zero lift AoA of an asymmetric airfoil will be zero."

Que paso?

peter kent
14th Oct 2012, 12:50
That's right Italia.

Lyman
14th Oct 2012, 15:13
Hi peter kent

I rebel against illogic, and squishy nomenclature. The potential for mistaking a value of lift should not be dependent on a formula that is merely an arbitrary definition.

At the point where an airfoil begins to develop lift becomes my datum. Zero means zero, or should. Just a quirk of mine, but not just me. It is an accepted datum in the industry,

There should not be confusion, and associating "Zero" angle of attack with actual work does just that. I prefer to let the design drive the nomenclature, not the reverse.

No problemo, peter :ok:

peter kent
14th Oct 2012, 15:44
Hi Lyman,
I apologise for being sloppy in my post. I was answering Italia on the J58 question he had addressed to me. I know nothing worth contributing on lift unfortunately.

peter kent
14th Oct 2012, 17:33
Perhaps someone can help with this one.

My brain generally reels reading the lift posts but I do have a question on what the authors are saying below.
The relevant bit is
"The BE is a statement of the conservation of energy. For it to be applied the system must be in equilibrium and no energy added to the system.... A great deal of energy is added to the air.... A 250 ton plane is doing a lot of work to stay in the air."
and the crucial statement:
"We have shown that the pressure and velocity of the air over a real wing in flight are not related by BE."

http://home.comcast.net/~clipper-108/Lift_AAPT.pdf

This seems at odds with people who use Bernoulli to trade off static pressure with velocity when designing real aircraft. These professional plane makers mention no qualifiers as regards work transfer.
Just one example
'Ch 10. The relation between supervelocity and pressure coefficient'
in a book bursting at the seams with Cp plots and much, much more for all airliners from DC-8 on and written by a Fokker man.

Aerodynamic Design of Transport Aircraft (http://www.scribd.com/doc/23956816/Aerodynamic-Design-of-Transport-Aircraft)


So shouldn't the college aerodynamics professor author at the top of the post be preparing students to go into industry?

italia458
15th Oct 2012, 21:21
Lyman..

Italia: "The zero lift AoA of an asymmetric airfoil will be negative. The zero lift AoA of an asymmetric airfoil will be zero."


Zero lift AoA is a geometric AoA measurement, meaning it's the angle between the chord line and the relative wind. Zero lift means that the airfoil is producing no net lift force in either the positive or negative direction - ie: there is not net deflection of the air either up or down.

The zero lift AoA for an asymmetric airfoil will be negative (ie: -3 degrees), the zero lift AoA for a symmetrical airfoil will be zero degrees.

italia458
15th Oct 2012, 21:48
Peter...

I briefly read through that paper and here are my thoughts on it. That paper seems to be written to just talk about false theories of lift. I can't speak for what the author's thoughts were but it seems he is passionate about correcting errors in popular theories of lift and he might have given the impression that Bernoulli isn't really responsible for lift... which isn't exactly correct.

This is one that he mentioned: Incorrect Lift Theory (http://www.grc.nasa.gov/WWW/k-12/airplane/wrong1.html)

I'd recommend just going through the other wrong theories on that NASA site.

I find people, naturally, want to find THE responsible thing for an event. If someone was killed, they want THE murderer. But in both cases, it isn't just one thing or person that is responsible for an event. Lift happens. We can see its effects, and the science of aerodynamics and physics studies it to try to understand how to manipulate lift forces favourably (for people to get to England in less than 2 weeks!). The thing is, no body really knows why lift happens. It can be explained why up to a certain point but then it breaks down pretty quickly, however, we do know quite a bit about how lift is created and to explain different aspects of it we have different laws and theories. There is no ONE theory that explains it all. Newton can describe how the forces that are created by the wing will actually lift it, Bernoulli helps explains the pressure differences around the wing, coanda/reynolds numbers/friction help explain how air is bent and how it reacts to different inertial and viscous forces, Kelvin's circulation theorem explains the circulatory flow around the airfoil in flight, etc. They all explain a little piece of the puzzle.

What this paper's goal seems to be is to let people know that Bernoulli doesn't explain EVERYTHING about lift, etc. I was taught the equal transit time theory (which is incorrect) when I was first learning and I notice it in books and have seen other instructors teach it. I think all these incorrect theories come about because people who aren't qualified to do so are 'simplifying' these theories so that it's intuitive and easy to understand for themselves and their students. The problem is that a lot of things aren't intuitive unless you have an understanding of physics and have studied these aerodynamics problems. Other instructors believe that EVERYTHING can be simplified and explained easily, which is not true. There is only so much simplifying that can be done before you end up with an explanation that is partially or completely wrong.

Does that provide any clarity to the paper?

FlightPathOBN
15th Oct 2012, 22:50
If the wing section at AoA of zero degrees produces no lift, then the lift associated with a wing section must be associated with Newtonian physics rather than Bernoulli gas laws...

Actually, IF the wing section did produce lift, with the current thrust provided, the wing section would be near a negative AoA, attempting to contain lift... (ie even at a negative AoA, the top surface is still longer than the bottom, hence a negative AOA would push the ac forward...)

Lyman
15th Oct 2012, 23:28
Italia. You realize the shaded box in your post is me quoting you? You claim two LIFT values for one AoA. The cross section for any chord is not relevant.

Hence the confusion. Zero Lift cannot be other than 0 AoA. Regardless of shape.

Asymmetric, Symmetric, flat plate, etc. It is not related to cross section.

Also reread your posit, AoA cannot be both negative and zero at zero lift. Any AoA other than zero produces lift, though in one case it would be inverted.

Do you not see this? Angle of Attack describes an "attack" angle, eg "other than "ZERO" Whether the lift is up or down, there is lift, hence other than zero AoA. To say otherwise draws attention to a deliberately arbitrary Incidence.

Zero AoA can be replaced with "LIFT NEUTRAL" if you wish.....

Please assume velocity sufficient to create lift....initiating....

FlightPathOBN.....

'If the wing section at AoA of zero degrees produces no lift, then the lift associated with a wing section must be associated with Newtonian physics rather than Bernoulli gas laws...'

muchas gracias.....

To validate Bernoulli requires something for nothing.... 0 degree angle of attack producing LIFT, in other words. Bernoulli's "accelerated upper flow" violates Newton's first LAW. Failing "accelerated", the argument defaults to "it travels further". No, it does not..... separated airflow needn't (and does not) arrive to meet at the trailing edge......

A Squared
15th Oct 2012, 23:38
At the point where an airfoil begins to develop lift becomes my datum. Zero means zero, or should. Just a quirk of mine, but not just me. It is an accepted datum in the industry,


OKay, so attempting to decipher your gibberish, apparently you are trying to claim that the only true AoA is that measured relative to the zero lift axis of the airfoil. Certainly there are occasions when that is convenient. But there in other contexts in which is is more convenient to express AoA as relative to the physical chordline of the airfoil, which can be seen and measured using ordinary measuring instruments. Doesn't really mater as long as the terms are defined clearly, but you'll find that the latter is a lot more common than the former, regardless of how firmly you you are convinced of it's superiority. Oddly, you seem unable to wrap your own mind around the two, as evidenced by your own words: Even an asymmetrical profile does nothing without AOA. At zero AoA its lifting force is downward....

If we accept your definition of AoA as being the angle relative to the neutral lift axis, an airfoil can hardly have a nonzero lift force at zero AoA, now can it?

I rebel against illogic, and squishy nomenclature.

If that is indeed true, it would appear that your own posts would be an excellent starting point for your rebellion.

A Squared
15th Oct 2012, 23:49
If the wing section at AoA of zero degrees produces no lift, then the lift associated with a wing section must be associated with Newtonian physics rather than Bernoulli gas laws...


Ummm, no. Beginning by defining angle of attack as the angle relative to the zero lift axis, then attempting to draw conclusions from the fact that the lift at a zero angle of attack is relative to the zero lift axis is ........wait for it............... zero (surprise!!!!!) is nothing more than a very obvious exercise in circular reasoning. Illogic, if you will.

Lyman
15th Oct 2012, 23:49
To clear it up partially, The second quote is true only if 0 angle of attack produces Lift, as per Italia's posit. No need to get hostile...

Look, the quote above 149 is from FlightPath OBN, please keep your quotes accurate. Your response to it is unintelligible.

You are completely disorganized, quoting one person as though someone else, etc.

I would ask you to correct your post, please, and utilize correct attribution.

Thank you.

A Squared
16th Oct 2012, 00:01
Look, the quote above is from FlightPath OBN, please keep your quotes accurate.

Ahhh, and so it is. But then, in your post, you quote him, and thank him for bringing up that point and agree with it, amd build from it in your own words.

Don't try to distance yourself from that thought now.

Lyman
16th Oct 2012, 00:05
It is your response that requires distance, with respect. I do agree with FPOBN, would you like reference?

A Squared
16th Oct 2012, 00:21
I do agree with FPOBN,


Which was exactly my point, you agree with the obviously circular reasoning.

would you like reference?

Reference for what? The fact that lift, at zero angle of attack, when defined as the angle which produces zero lift, is zero?

No thanks, I think I can work though that on my own.



Incidentally, I've edited the quote attribution, as you requested

Lyman
16th Oct 2012, 00:27
It is not circular, how so? Bernoulli requires Lift at 0 AoA to sustain, if there is none, Lift is explained via Newton. There is a reference in the paper posted by peter kent. Did you catch it? The paper also proves that upper surface airflow velocity is less than the lower airstream. substantially less. Additionally, if Bernoulli applies on a real wing, the upper surface would be shaped like half a circle.



Thanks for the re-do.....

A Squared
16th Oct 2012, 00:50
There is a reference in the paper posted by peter kent. Did you catch it? The paper also proves that upper surface airflow velocity is less than the lower airstream. substantially less.

Wrong Again!

Yeah, I caught it. the trouble is, it says exactly the *opposite* of what you claim it says.

Here's what the paper says (verbatim quote) The greater the lift the greater the different (sic) in arrival times at the trailing edge with the air going over the top of the wing arriving considerably before the air below the wing.

Notice that part in red? The air over the top of the wing travels faster than the air below.

the author is merely addressing the "equal transit time" myth, which pretty much everyone understands is a myth.

Lyman
16th Oct 2012, 00:53
My recall is the upper air arrives later, I'll look.

A Squared
16th Oct 2012, 00:59
My recall is the upper air arrives later, I'll look.

And herein lies the problem: most of what you "know" is wrong.

On a cambered wing that is producing upward lift the airflow over the top is faster than the airflow over the bottom. This is one of the basic facts that is not in question and is demonstrated unambiguousky by more than a century of wind tunnel data.

You don't have even this most basic piece of information correct. yet you presume to lecture others on the *truth* of aerodynamics.

A Squared
16th Oct 2012, 01:09
It is not circular, how so?

Probably a complete waste of my time but here goes:

There are several valid ways to define angle of attack.

You chose to define angle of attack as the angle relative to angle where it produces zero lift.

There's nothing wrong with that definition as far as it goes.

But.......


Inevitably, if you define angle of attack in this manner, the lift at zero angle of attack is by definition, zero. This is a result of your definition of angle of attack, not a demonstration of some overriding physical principle, yet you attempt to use it as "proof" of some principle.

If you can't follow how this is nothing more than circular reasoning, I can't help you.

Lyman
16th Oct 2012, 01:35
It seems I got well ahead of myself, I read it wrong, and didn!t stop to think about it, my apologies. Also for my tone, it was insufferable.

The paper makes reference to "effective angle of attack", which I mean to take as the zero lift position for any lifting surface, regardless of shape, is this how you read it? I'll leave some room here for others and will read for now.

Thanks.

italia458
16th Oct 2012, 02:56
This looks like a whole bunch of non-sense going on here!

Lyman...

Italia. You realize the shaded box in your post is me quoting you? You claim two LIFT values for one AoA. The cross section for any chord is not relevant.

:ugh: Of course I realize that's you quoting me...

Re-read my post where I defined the AoA terms that I was using and where I provided references to further expand on my definition. It might take time to sink in but what I said makes sense.

italia458
16th Oct 2012, 02:59
FlightPath...

If the wing section at AoA of zero degrees produces no lift, then the lift associated with a wing section must be associated with Newtonian physics rather than Bernoulli gas laws...

That's a false premise.

In earlier posts I stated a few definitions of AoA... I'm not sure you have the same understanding of those definitions as I do.

roulishollandais
16th Oct 2012, 15:44
I learned a lot seeing the old "polaires de Prandtl"...:ok:
they are the Cz(Cx) closed graphs at a definite airspeed.
Would anybody know where we could find them on the Web ? Thanks

italia458
16th Oct 2012, 17:22
Lyman...

I have no idea what you're trying to say... I don't think you understand the definitions that I've already mentioned: geometric angle of attack and absolute angle of attack. It's generally accepted that, unless otherwise specified, when ever you mention AoA you are talking about the geometric angle of attack. If you did understand them you'd either understand what I've already said or you'd be able to specifically point out an error I might have made.

From the paper in discussion, the notion of "Zero lift defines the effective angle of attack" would appear to dismiss Bernoulli.

Please explain why you believe this definition dismisses Bernoulli. Effective angle of attack is yet another definition that is completely different from geometric and absolute. Do you know what effective angle of attack means?

peter kent
18th Oct 2012, 22:41
Italia,
Thanks for a good overview of the complexities of trying to get to grips with the way air behaves.

I think the author shunned Bernoulli's relation completely by making a revolutionary claim that I have never seen before. It is not one of the regular myths/misapplications that he is repeating. Here's my way of explaining what I think he was saying, and I have repeated his statement at the end:

Focussing on just the wing surface, ie its own map of local curvature variations and static pressure distribution that goes with the curvature.
Knowing the free-stream condition and a measured static on the surface you can calculate the vel at the same point using conservation of energy, ie with Bernoullis relation.
If we now drop a ram air turbine into the flow we can no longer use the relation for points U/S of the turbine and D/S because of the work extraction in between.
We can, of course, use the relation for all points D/S once we have redatumed, if you like, with a new lower total pressure.
Now, just as we say the whole of the lift force occurs from the wing surface pressure distribution we also say the whole of the energy transfer to the air also occurs only on the wing surface.
So energy is not conserved in the airflow over the wing any more than it was with the RAT power extraction.

"As energy is not conserved the Bernoulli relation cannot be applied to airflow round a wing in flight."

Nobody else throws it out for this reason. It's a new one as far as I can tell.

Can you make any sense of it?

Volume
19th Oct 2012, 09:10
This is one that he mentioned: Incorrect Lift Theory
Of course that theory is completely incorrect, it assumes that if the airsteam ahead of the airfoil is parallel to the airstream aft of the airfoil, then lift is produced. Of course bernoulli fails to explain a wrong assumption...

As energy is not conserved the Bernoulli relation cannot be applied to airflow round a wing in flightInteresting aspect. However, as long as we ignore friction (which we usually do in all those theories), the force generated ("Lift") is perpendicular to the direction of movement, hence there is no work done, a force perpendicular to a movement does not produce work, hence energy is conserved. The same applies to any element of the wing surface, pressure (hence force) is produced perpendicular to the local surface, and therefore perpendicular to the streamlines, so along the streamlines energy is conserved.
But interesting to think about, how a Rat can then extract energy... Or how a propeller can inject energy with bernoulli (constand energy along a streamline) still being valid...

italia458
19th Oct 2012, 18:12
Peter...

I just read the whole article word for word and I agree with his description of lift. It has added a new perspective to my understanding of lift and to me it makes sense.

I'm not sure exactly what you're trying to say but I think it has to do with not seeing where/how energy is added to the system?

Think of the air standing still with reference to the ground and an airfoil comes passing through it. If you look at the air you will see that it gets accelerated from rest (zero velocity) to a positive velocity (speed and direction of movement) as the airfoil passes by. Energy has just been added to the system. So to create lift, energy has to be added to the system to get the air to flow (start moving) around the wing and for downwash to be 'created'. That downwash is then what forces the wing upwards. There are two ways to add energy into the system (ie: accelerating the air): by adding propulsive power to the airplane which gets directly transferred to the airfoil and which transfers that power (energy) to the air, or you have no engine and you use gravity as the 'engine' as gliders do. In a glider, assuming that the air is still and there are no thermals, the glider (while flying at a constant velocity) will have a constant horizontal and vertical component of velocity. That vertical component of velocity is what is adding energy to the system and the factor responsible for that vertical component is gravity.

With regard to Bernoulli: he's basically saying that there is energy added to the system and Bernoulli describes the conservation of energy for a system (where energy is neither added or subtracted from a system) so Bernoulli doesn't apply. Bernoulli is a mathematical description of an effect that happens in a closed system. Bernoulli doesn't say this: Think of two adjacent streamlines with different speeds. Since these streamlines have different velocities forces between them trying to speed up the slower streamline and slow down the faster streamline. The speed of air at the surface of the wing is exactly zero with respect to the surface of the wing. This is an expression of viscosity. The speed of the air increases with distance from the wing. Now imagine the first non-zero velocity streamline that just grazes the highpoint of the top of the wing. If it were initially to go straight back and not follow the wing, there would be a volume of zero velocity air between it and the wing. Forces would strip this air away from the wing and without a streamline to replace it, the pressure would lower. This lowering of the pressure would bend the streamline until it followed the surface of the wing. The streamlines are bent by a lowering of the pressure. This is why the air is bent by the top of the wing and why the pressure above the wing is lowered. This lowered pressure decrease with distance above the wing but is the basis of the lift on a wing. The lowered pressure propagates out at the speed of sound, causing a great deal of air to bend around the wing.

That is essentially what is happening on a real wing. Like others have said, you need to have viscosity to produce lift. If the molecules weren't 'connected' to each other then they would have no 'communication' between streamlines and you could end up with a condition where one streamline has a velocity of 10 m/s and the two adjacent streamline velocities are 200 m/s and 3000 m/s, all while travelling in a straight line! If that were the case you would not have the air bending, and you would therefore not have any downwash.... meaning no lift.

roulishollandais
19th Oct 2012, 18:39
Definition of independant "streamline" please....:}

italia458
19th Oct 2012, 19:39
Definition of independant "streamline" please....

Definition of Streamlines (http://www.grc.nasa.gov/WWW/k-12/airplane/stream.html)

henra
19th Oct 2012, 19:43
"As energy is not conserved the Bernoulli relation cannot be applied to airflow round a wing in flight."

Nobody else throws it out for this reason. It's a new one as far as I can tell.

Can you make any sense of it?

Interesting thought!
However the problem of energy conservation only significantly applies in the lower parts of the boundary layer of the wing. If you look at the speed and thus energy above the immediate boundary layer it will be close to the free stream air speed and thus you still have accelerated air speed and therefore reduced static pressure.
The streamlines above the boundary layer might not be perfect anymore and you might loose some lift compared to the theoretical lift with no boundary layer. But the general effect (Air accelerated creating low pressure and thus lift) will still be there.

Therefore my appreciation of this is that the effect described in the article might spoil the result of a calculation based on Bernoulli under ideal conditions but it does not prove that Bernoulli doesn't explain lift. At least my picture is not completely ruined (yet).

italia458
19th Oct 2012, 20:06
If you look at the speed and thus energy above the immediate boundary layer it will be close to the free stream air speed and thus you still have accelerated air speed and therefore reduced static pressure.

True. But saying Bernoulli is responsible for this is exactly what this paper was highlighting as a 'myth'.

But the general effect (Air accelerated creating low pressure and thus lift) will still be there.

Not true. Lift is directly related to the downwash of air. The article also explained that the acceleration of air DOES NOT create the low pressure - the low pressure causes the acceleration of the air.

If you have air flowing around an airfoil that is producing lift you will have a relative high pressure on the bottom and a relative low pressure on top. If you are saying that since there is a low pressure on top and high flows to low, the wing will get pushed/sucked up - that's not really true. Air has to get deflected to create 'lift'. If air is not deflected by the airfoil, there is no net force and therefore, no lift. That's also why spaceships need retro rockets in space to maneuver because there is no surrounding atmosphere where a change in shape of the spaceship would produce a net force - they need to apply a net force by expelling mass at a high velocity (rocket). Putting those two together, if there is a low pressure, the air will flow in the direction of the low pressure - which is upwards. If the air flows upwards, according to Newton's 3rd law, there will be an equal and opposite down force on the wing. That's the opposite to lift!

Lyman
19th Oct 2012, 20:21
Howdy Italia458

Just a comment, see what you think.

-" the low pressure causes the acceleration of the air." Not the way I learned it, for air flows into low pressure as the result of inhabiting a higher pressure in the locale. The air is acceleratd by the higher pressure which is created by, work.

Perhaps pedantic, though I don't think so. Professor Nicholson demanded that in Nature, there was no "pull" only push.

I see you like the paper. I especially am drawn to "air rest frame" and "wing rest frame". My apologies if I am thick....

henra
19th Oct 2012, 20:52
Air has to get deflected to create 'lift'. If air is not deflected by the airfoil, there is no net force and therefore, no lift.



I take it you are a 'Newtonian' :E

But if we assume we have high pressure on the lower side and low pressure on the upper side I fail to see why simple physics shouldn't apply:
F=p*A, p being the pressure differential and A the wing area.


...if there is a low pressure, the air will flow in the direction of the low pressure - which is upwards. If the air flows upwards, according to Newton's 3rd law, there will be an equal and opposite down force on the wing. That's the opposite to lift!One word: Inertia.
Somewhere on the first pages of this thread I commented on how Bernoulli and Newton are somewhat linked.

That is why, if you have an airfoil where the camber at the TE points downward , the air molecules will continue downward even behind the Trailing Edge. Therefore you have a bigger 'expansion' and thus higher acceleration over the wing, reducing the pressure according to Bernoulli. This same effect gives you a bigger mass stream deflection according to Newton.
In my eyes Bernoulli and Newton are not really conflicting. they are different ways of looking at the same phenomenon, that is causing the lift.
This effect itself must be somehow linked to the behavoíour and interaction of the molecules in the air, because lift requires viscosity and mass.

italia458
19th Oct 2012, 20:53
Lyman...

Are you familiar with F=ma? If you are, you'll know that for a mass (ie: a book, car, computer, apple or air) to be accelerated, there must be a net force. A net force means an 'excess' or imbalance in the forces on the object. For example: When a book is resting on the table, there is no net force on the book, therefore the book is not accelerating. Since the start velocity was zero and there is no acceleration, the book will continue to lie on the table forever... unless there is a net force applied to it.

Pressure is essentially a type of force - think of it as the source of a force. This could be water pressure (force) from a firehose that knocks you off your feet. So for the air to be accelerated, there must be a net force applied to it. Just as a clarification, an acceleration is required to change either direction OR speed since acceleration is related to velocity and velocity is a vector quantity (meaning it has both a magnitude (speed) and direction). So in the case of the air flowing over the wing, it is the low pressure (force) that is applied to the air that accelerates it. If the air runs into a higher pressure area (like as it passes underneath the wing, close to the surface) it will experience a net force and accelerate in a negative direction (aka: slow down).

In all cases here we're dealing with static pressure and not dynamic pressure so the actual speed of the air does not change the pressure measured - don't think Bernoulli here! For example: most test airplanes will have a very long probe attached to the nose of the airplane to measure free stream attributes - this is before the air is affected by the airplane. It doesn't matter how fast the airplane flies, the static pressure measured will be the same as long as the airplane is flown at a constant pressure level such as FL050.

The air is acceleratd by the higher pressure
Professor Nicholson demanded that in Nature, there was no "pull" only push.

To me, all that is is relativity. High pressure pushes, low pressure pulls is the same thing. It's a property of fluids that they try to equalize so that there is no net internal force applied by itself. The reason there is higher pressure at sea level and lower pressure as you climb is essentially just because there is gravity.

italia458
19th Oct 2012, 21:14
I take it you are a 'Newtonian'

I wouldn't call myself a scientist but I have essentially the same 'beliefs' as they do. I'm not on any one side, I'm all about exploring what the real answer is, regardless of what it is.

http://cdn4.explainthatstuff.com/airfoil-wind-tunnel.jpg

Disregard the arrows in that picture. It might help to visualize the airfoil like in the picture above, except start with the airfoil pointing vertical (perpendicular to the air flow). With the airfoil stationary to the air, you'll measure that the static pressure around the whole airfoil is exactly the same as the ambient static pressure. As soon as you start moving that airfoil in one direction (not the band :}) you'll see that a low pressure area develops on the side opposite to the direction of movement. The faster you move the more pronounced is this area of low pressure. You'll notice that the air wants to rush in and fill that void. That is exactly what's happening when air is flowing over the wing in normal flight - there is a slight void created (not as dramatic as the one you're visualizing) and the surrounding air expands to fill it, which decreases pressure and accelerates the flow.

Now that you're experimenting with that concept I should point out that at these extreme angles of attack the explanation of the origin of the force which pushes the airfoil upwards is different. At very high angles of attack there is a noticeable amount of lift that is generated by the deflection of air off the bottom of the airfoil. However, in normal flight, these extremes are not reached and therefore it is not really a valid explanation of lift of an airfoil in flight.

Lyman
19th Oct 2012, 21:22
Thanks Italia, I do remember that...

Acceleration. May I leave that for now? I see the wing as an engine. It is an air compressor, a device that increases air pressure faster than the air can escape. This happens dynamically, and pressure is created, then maintained, in an open area.

The viscosity and pressure keep the air from moving fast enough to "escape". For simplicity, the area below the wing is high pressure, above the wing low.

The upper streamline and the wing itself trap the low pressure zone. From the link, "No Mass can penetrate a streamline". As long as the low is confined, the high maintained, the high pressure air wants to enter the low, it has no time to escape around the leading or trailing edges, this works as a dynamic lifting system, the system is created and destroyed thousands of times each Second.

I do apologize if this sounds stupid... But I am an amateur....

bookworm
19th Oct 2012, 22:01
With regard to Bernoulli: he's basically saying that there is energy added to the system and Bernoulli describes the conservation of energy for a system (where energy is neither added or subtracted from a system) so Bernoulli doesn't apply. Bernoulli is a mathematical description of an effect that happens in a closed system.

Consider the dynamics happening on a pool table. If we simply ignore friction, we can make extremely accurate predictions of the behaviour of the balls using the conservation of momentum and energy. And that works because friction is low compared to the other aspects of the dynamics.

But hold on! The balls are rolling, not sliding. So there must be friction, right? Oh dear. The laws of conservation of momentum and energy can't possibly be used to explain the motion of the balls because it's no longer an energy conserving system. And yet still, we manage to direct the balls to the pockets with unfailing reliability. How can that be?

Consider the dynamics of a wing. If we simply ignore viscosity, we can make extremely accurate predictions of the behaviour of the wing using Bernoulli's theorem. And that works because viscosity is low compared to the other aspects of the dynamics.

But hold on! We need some viscosity for the Kutta condition and to explain the boundary layer etc.. So there must be viscosity, right? Oh dear. Bernoulli's theorem can't possibly be used to explain the motion of the air because it's no longer an energy conserving system. And yet still, we manage to predict the lift coefficients with unfailing reliability. How can that be?

peter kent
20th Oct 2012, 00:03
bookworm,
I had a suspicion that the authors (Anderson and Eberhardt "Understanding Flight') were trying to throw something out because it wasn't accurate to the nth degree.
After all, the planemakers themselves use it OK.
Enough of them have written books based on their time in industry showing it's good enough for them.
eg Richard Shevell (Douglas) "Fundamentals of Flight"
Ed Obert (Fokker) "Aerodynamic design of Transport Aircraft"

italia458
20th Oct 2012, 00:52
Lyman...

What you're saying makes sense.. but not to create lift - it makes sense to accelerate the airflow and deflect it downwards. It's the downwards velocity that creates an upward force on the wing that lifts it. The pressure isn't the direct cause to the lift.

In the article he describes how an airplane flying over a big scale would indicate the weight of the airplane - the earth does not get lighter when the plane takes off. It seems like you believe that the 'suction' created by that void on top of the wing is responsible for sucking the wing upwards. If you're sucking in air, you need to displace it somewhere - where are you displacing this air that you're 'sucking' up?

Bookworm and Peter...

It's not that Bernoulli is wrong with regard to pressures and velocities - it is correct that as the flow over the wing is accelerated, the static pressure will drop. The author seems to have a big deal with saying Bernoulli is involved for a wing in real life when it's clear that Bernoulli is for a closed system and it's also clear that lift in real life is an open system with energy added.

The measurements of the static pressure and velocity is important to plane makers because they can calculate lift/circulation around an airfoil and determine how the airfoil will perform in real life, with adjustments for accounting for viscosity (Computational Fluid Dynamics). I don't think the authors mean to say that, what the aerodynamicists that work on designing planes do is, ignore Bernoulli because it's not applicable - it sounds like the author wants to argue that a better 'overall' description of lift should focus on Newtonian laws, as it once did.

That's my take on it.

Lyman
20th Oct 2012, 01:24
Italia...

"It seems like you believe that the 'suction' created by that void on top of the wing is responsible for sucking the wing upwards."

I have said, There is no pull, only push. How then do you read me thinking the air "pulls"? You have said the low-pressure accelerates the air, that is "suction".

The paper dismisses Bernoulli as supporting acceleration causing low pressure. The paper supports low pressure causing acceleration. I believe both are incorrect. Acceleration is caused by high pressure at the leading edge escaping aft over the wing, as I see it.... In any case, the major portion of the high pressure migrates below the leading edge, into a wedge shaped RAM (remember RAM:ok:)? The angle of the wing deflects airflow down, causing uplift, and this downwash meets the upper streamline and is increased thereby...

I understand that I may be a hard read, it frustrates me also. There are some medical issues...

I cannot thank you enough for your patience.....

Owain Glyndwr
20th Oct 2012, 08:55
This discussion has devolved into a debate on what Anderson and Eberhardt’s (A&E) article ‘Understanding Flight' really means. Having read it two or three times now it seems to me that it is a bit like the curate’s egg – ‘Good in parts’

They go to some pains to debunk the familiar “equal transit times” explanation which is fair enough because it doesn’t hold water, but that is because the basic assumption of equal transit times is wrong, not because of the subsequent attempt to link the undoubted fact that air flows faster over a wing upper surface than the lower generates a differential pressure which can be (in principle) calculated using Bernouilli’s equation. Their arguments on the invalidity of Bernouilli when applied to flow around a wing are tilting at strawmen I think.


Another fundamental problem with this description is that the air’s pressure and speed are not related by the Bernoulli equation for a real wing in flight! The Bernoulli equation is a statement of the conservation of energy. For it to be applied the system must be in equilibrium and no energy added to the system. As you will see in the discussion below, a great deal of energy as added to the air. Before the wing came by the air was standing still. After the passage of the wing there is a great deal of air in motion. A 250-ton jet at
cruise speed is doing a lot of work to stay in the air. Much of the fuel that is burned is adding energy to the air to create lift. Thus the Bernoulli equation is not applicable
I disagree. The energy added is used to overcome a side effect of lift generation – drag.

Picture in your mind several wings: an asymmetric wing in normal flight, the same wing in inverted flight, a symmetric wing and a flat plate. For each one, an orientation into the wind can be found which gives zero lift. We call this orientation the zero effective angle of attack. Now if one were to measure the lift of these wings as a function of the effective angle of attack, the results for all of them would be similar
Nobody will argue with that. Put another way it says that the lift curve slope of a wing of infinite aspect ratio is independent of camber.

But the consequence of using this definition of AoA is that each and every wing section has a different datum, making it impossible, or at least very difficult, to compare the characteristics of various sections – not a great idea!

There is another mistaken description of lift, which we will call the wrong-Newtonian description of lift, although those that teach it just call it the Newtonian description of lift. This description of lift states that diverting air down produces lift, and that lift is a reaction force. This part is true. Unfortunately, in the wrong-Newtonian description of lift the air is diverted down by impact with the bottom of the wing. .....
Although there is a little of this kind of lift for most wings, it is minimized for efficient wings. The amount of air impacted by the bottom of the wing is far too small to account for the lift.

No problem there either


Yet another common description of lift is that of circulation theory. Here the air is seen to rotate around the wing. This is sometimes used to explain the acceleration of the air over the top to the wing. There is a great deal of jargon, such as "starting vortex" and "bound vortices", associated with this description. Circulation theory is a mathematical abstraction useful and accurate for aerodynamic calculations. Mathematically, circulation is a non-zero curl in the airflow in a closed line integral around a wing, which
is simply a statement that the wing bends the air.

For sure circulation theory is usually expressed in complicated math, but it can also be explained in plain English (or even American!). Way back in post #33 of this thread I gave a url. for such an explanation. It does a lot more than just state that the wing bends the air.

In brief, the lift of a wing is a reaction force and is proportional to the amount and vertical velocity of air is diverted from the horizontal to the vertical, with almost all of the air diverted from above the wing.
Again, no problem with this as an overall explanation, but it doesn’t really tell us much about how it all happens. A&E go on to say that

Lift = mdot * vv

Where mdot is a mass flow rate and vv is the vertical downwash velocity imparted by the wing measured in the air’s rest frame. That is a trivial statement unless we can understand a bit more about the two terms.

They suggest:

We would first like the reader to view the wing as a kind of "virtual scoop" as illustrated in figure 5. The amount of air intercepted by the wing is related to the lift distribution along the wing. The shape of the virtual scoop is half of an ellipse with the major axis equal to the wingspan and the minor axis proportional to the chord length (distance from leading to trailing edges) of the wing. The air intercepted is diverted down with the highest downward velocity near the wing and the deflection speed tapering to zero as the distance above the wing increases, as shown in the figure. This is not intended to imply that there is a real, physical scoop with clearly defined boundaries, and uniform flow. But this visualization aid does allow for a clear understanding of how the amount diverted air is affected by speed and density
The amount of air intercepted by the scoop, mdot, is proportional to the
• area of the wing
• wing’s speed
• air’s density
To a good approximation, neither the angle of attack nor the load on the wing affects the
amount of intercepted air.

In one respect this is wrong. mdot is the flow leaving the TE and is proportional to wing span not wing area. We still have no idea how much air is affected though. A&E calculate that the deflected air might be drawn from as much as a semispan above the wing, but this is based on some assumptions rather than any definite scheme of things.

So far as I can see, apart from saying that it is proportional to AoA and airspeed they give no guidance on how the vertical velocity is generated – a fundamental piece of knowledge so far as our understanding of lift generation is concerned.


Moving on to their strictures on the (mis)use of Bernouilli, they are of course correct when they say that the general application should be:

Static pressure + 0.5 rho*V^2 = Total pressure

Their argument is that if energy is added to the flow then total pressure will increase and Bernouilli’s equation will be invalidated. Equally true of course if energy is extracted from the flow. But let us look a little deeper.

There is abundant evidence from wake survey experiments that total pressure is not constant behind a wing producing lift, so in broad terms they are right, although not because energy is added – rather it is subtracted. However, it is also true that this loss of total pressure is confined to a small area just behind the TE – in the wing wake in fact.

The NASA site that Italia 458 referenced to define streamlines says:


A streamline is a path traced out by a massless particle as it moves with the flow. ........ Since there is no normal component of the velocity along the path, mass cannot cross a streamline. ............. We can use Bernoulli's equation (http://www.grc.nasa.gov/WWW/k-12/airplane/bern.html) to relate the pressure and velocity along the streamline. .............. Since no mass passes through the surface of the airfoil (or cylinder), the surface of the object is a streamline.
The wing surface may be a streamline, but since the streamwise velocity is zero everywhere on the wing surface applying Bernouilli there would be silly. Close to the wing the streamwise velocity increases steadily through the boundary layer, but in a turbulent boundary layer (as exists over 90% plus of the wing’s surface) there is a constant, if random, exchange of mass from the high energy outer regions towards the regions close to the surface. It is this energy transfer that permits turbulent boundary layers to accept higher adverse pressure gradients before separation. However, this exchange of mass means there can be no streamlines inside the boundary layer and consequently Bernouilli’s equation cannot be applied there.

At the outer edge of the boundary layer (where there is no more mass transfer) there will be a bounding streamline and from this point out Bernouilli may be applied. This is confirmed by all those wake surveys, which show that outside the wing wake the total pressure is constant.
What does this mean for the application of Bernouilli to the flow around a lifting wing? It means that it can be used to calculate pressures and velocities around a shape that is close to, but not exactly the same as, the basic wing. This does not mean though that the equal transit time explanation can be retained!

I think I can safely say that none of this bothers practising aerodynamicists who are perfectly happy to use pressures measured on the wing surface and go from there via 0.5rhoV^2 to get to wing loading.

Then there is the bit:

In other words, the pressure difference drives the acceleration of the air, not the other way around.
Yup! But I don’t see it as any pressure difference along a streamline. Think in their air-rest frame. If the air is obliged to follow a curved path as the wing passes through it there must be a centripetal force making it do so. This has to be some sort of pressure differential. But the total pressure of the air at rest is equal to the ambient static pressure. The pressure differential therefore has to come from a drop in pressure at the wing surface (or more strictly I suppose at the outer edge of the boundary layer). Since Bernouilli applies at the outer edge of the boundary layer this drop in static pressure will be accompanied by an increase in velocity. Pressure difference across streamlines is the driving force.

Rather grudgingly, A&E say:

Although circulation theory can be used for accurate calculations of lift, it does not give a simple, intuitive description of the lift on the wing. We have also shown that the pressure and velocity of the air over a real wing in flight at not related by Bernoulli’s equation. Newton's laws hold without exception for both subsonic and supersonic flight, and can be used to yield an understanding of many concepts without complicated mathematics
From my pov. Newton’s laws as expressed by A&E do give a simple quantification of the lift on a wing, but it is if anything oversimple. It tells us nothing of how lift is actually generated nor how it might be distributed over the wing. If that is all you want, then fine, but if you want a bit more depth then I can only refer you back to the Arvin Gentry article I referenced in post#33 – plain English but realistic and informative.

henra
20th Oct 2012, 08:59
So what we see is air entering a BIGGER area and expanding to fill the area, and hence, decreasing the static pressure. But Bernoulli says that if air in a closed system goes into an area of bigger dimensions the air will expand to fill the area and INCREASE static pressure! The difference being that air over an airfoil in flight is not in a closed system, energy IS being added to the system and that energy is added to the air.


Yes, expansion means increasing pressure. But increasing can mean increasing back to free stream static pressure. No discrepancy to the 'closed' system here. The same applies for the energy. Also in a closed system you will convert energy if viscosity and friction exists. Those two don't care much if there is a tube around. You still have boundary layer, speed gradient, etc.
So I still do not really see where the fundamental difference between a closed system and an open system lies that would conflict with the general principle.

The expansion happens as any gas is always looking for equilibrium. So does air. That makes gases try to fill voids.
Re: Push vs. Pull: Pull is the consequence of less Push on ine side than on the other.
Knowing that we don't live in a vaccum we consider any pressure lower than ambient pressure as Pull although technically it is simply less Push as ambient.
Still in an ambient pressure of 1013 HPa we generally consider anything below as Pull or Suction in daily life.

Lyman
20th Oct 2012, 12:20
That is fundamentally the point I try to make, the wing compresses air, and does so in such a way as to create a very real and bordered system. Because air has mass, and seeks to equilibrate, a dynamic force lifts the wing. The system is constantly being destroyed, and created.

The zone of low pressure above the wing is a byproduct of compression up stream, and as a result of process, cannot be said to be its initiator. Bernoullians want folks to look at the magic of low pressure, and at its captivating creation by "accelerating" the air mass, locally. Fine, so far, but there is no "magic", lift takes work, hard noisy work, and if one wants to lift, one must stay moving. One must look at Newton to describe the system, Bernoulli describes local artifacts not the system.

The 'held in place' cambered section diagram is the recipient of airflow, not its creator. A flat plate is no different, they both produce lift by compressing air at the leading edge, both above and beneath it.

"A and E", in their paper, completely destroy the arse-about paradigm that the "shape" (section) of an airfoil makes some kind of basic difference in the fundamental Laws that create Lift.

HazelNuts39
20th Oct 2012, 12:56
A&E calculate that the deflected air might be drawn from as much as a semispan above the wing, but this is based on some assumptions rather than any definite scheme of things.According to A&E, L = mdot*vv. Assuming that all the mass captured in the "virtual scoop" uniformly gets a downwash velocity vv, the power required for this is Pi=0.5*mdot*vv^2. Equating this to the power Pi=Di*V to overcome the induced drag Di, it can be shown for an elliptic lift distribution that the area of the scoop is equal to a circle with a diameter equal to the wing span.

Lyman
20th Oct 2012, 13:01
The entire circle, or half of it?

"it can be shown for an elliptic lift distribution that the area of the scoop is equal to a circle with a diameter equal to the wing span."

Owain Glyndwr
20th Oct 2012, 13:28
Hi HN39

it can be shown for an elliptic lift distribution that the area of the scoop is equal to a circle with a diameter equal to the wing span.But what about 2d wings (original point of discussion) or non-elliptic loading (usually the case)? Downwash not uniformly distributed?

Lyman
20th Oct 2012, 13:51
A very large flat plate surface is entering the atmosphere, it dips into the air to a point where it is gliding, and there is no air above its surface. What is holding this flat plate at constant altitude?

HazelNuts39
20th Oct 2012, 15:05
Lyman,

The entire circle. But of course the downwash velocity is not constant. It reduces with the distance to the wing asymptotically to zero at infinity.

Lyman
20th Oct 2012, 15:43
Howdy HazelNuts39

The scoop in the paper is oriented above the wing. For the area of the circle to repose above the wing plane, it would be flattened, and perhaps taller than one radius?

What is the descriptor for the area beneath?

The conclusion of the two authors...

"We have also shown that the pressure and velocity of the air over a real wing in flight are not related by Bernoulli’s equation."

Not only does airflow above the wing not create lift, it is detrimental to the work beneath it. It is after all, just additional mass to lift, with the aircraft....

It is the diminution of the airflow pressure above the wing that is a result of increase beneath. If the two are negatively related, how can both be positive?

roulishollandais
20th Oct 2012, 17:45
à Italia458, Owain Glyndwr,

We already rejected here these picture (despite coming from NASA).

Try to draw it at infinite...left and right, up and down, and 3-D of course, you have surprises !

The object of that thread is precisely to get a correct theory of lift, not this one !


http://www.grc.nasa.gov/WWW/k-12/airplane/Images/logo_nasa.gif (http://www.nasa.gov/)+ Text Only Site (http://www.grc.nasa.gov/WWW/k-12/airplane/stream.html#)
+ Non-Flash Version (http://www.grc.nasa.gov/WWW/k-12/airplane/stream.html#)
+ Contact Glenn (http://www.nasa.gov/centers/glenn/about/contact_us.html)
http://www.grc.nasa.gov/WWW/k-12/airplane/Images/title_find_it_sm.gifhttp://www.grc.nasa.gov/WWW/k-12/airplane/Images/spacer.gifhttp://www.grc.nasa.gov/WWW/k-12/airplane/Images/button_go.gif (http://javascript<b></b>:document.search.submit();)http://www.grc.nasa.gov/WWW/k-12/airplane/Images/nav_top_0_0.gif (http://www.nasa.gov/about/highlights/index.html) http://www.grc.nasa.gov/WWW/k-12/airplane/Images/nav_top_1_0.gif (http://www.nasa.gov/news/highlights/index.html) http://www.grc.nasa.gov/WWW/k-12/airplane/Images/nav_top_2_0.gif (http://www.nasa.gov/multimedia/highlights/index.html) http://www.grc.nasa.gov/WWW/k-12/airplane/Images/nav_top_3_0.gif (http://www.nasa.gov/missions/highlights/index.html) http://www.grc.nasa.gov/WWW/k-12/airplane/Images/nav_top_4_0.gif (http://mynasa.nasa.gov/portal/site/mynasa/index.jsp?bandwidth=high) http://www.grc.nasa.gov/WWW/k-12/airplane/Images/nav_top_5_0.gif (http://www.nasa.gov/about/career/index.html) http://www.grc.nasa.gov/WWW/k-12/airplane/Images/stream.gif

Owain Glyndwr
20th Oct 2012, 21:10
roulishollandais

It should have been obvious from my earlier posting that I wasn't trying to use the pictures to explain lift; merely using their entirely correct definition of a streamline.

Or do you have another definition of a streamline?

If you want an entirely correct theory of lift then read Arvin Gentry.

roulishollandais
20th Oct 2012, 21:38
Hi Owain,
Thank you for your answer. ... I reject the "idea" of streamline, as it is just an idea, and idea do not fly... It was an idea for the time of closed windtunnel! We are in 2012 .We are unable to continue the draw of the streamlines to their limits : beginning, end, the aircraft is flying inside an enormous balloon of same mean density that air around it... The biggest problem is not energy or pressure but information transfer...Dear Holy Shannon Help please ! Or let us find an other model than streamlines@@@@@_______

Lyman
21st Oct 2012, 04:46
"Quote:
they both produce lift by compressing air at the leading edge, both above and beneath it"


"Absolutely not. Lift, at its most basic, is created by the turning of the airflow."

And before the airflow can be deflected, it contacts the leading edge, which produces a high pressure "bubble".

Look, I am nowhere near the expert I perceive you to be, and I am profoundly interested in the discussion. Sending people off to the "library" strikes me as dismissive, and tends to smother the discussion, rather than enlarge it. I was scolded at one point by someone for looking at the paper and supporting it, who has now admitted it filled some holes, and he is a convert.

I don't smoke a pipe, nor join groups just to nod and affirm each others' bias.

I think these two guys have some elegant work on display, and honestly, I never liked Bernoulli.

Still air is not a pipe, Air is not a liquid, It is the wing that moves, not its medium, And low pressure is not created by acceleration of the medium.

"It is our hope that teachers will return to the basics and use Newton's laws to describe lift. Then students can explore flight in much more depth than was possible with the popular explanation using Bernoulli."

Owain Glyndwr
21st Oct 2012, 06:35
I reject the "idea" of streamline, as it is just an idea, and idea do not fly.
Better not tell that to a aerodynamicist. There are so many misconceptions posted on this thread that I've scanned pages from the bible of how lift is created, "Theory of Wing Sections" by Ira Abbott and Albert Von Doenhoff. Some heavy maths, but never the less the text should shed a little light.He just did :)

Abbot & von D is heavy on math. I've just located a better article by Arvel Gentry than the one I have been recommending which covers the same ground but without the maths. It talks about lift from sails, but it is all equally applicable to wings. It even describes how you get lift from a flying barndoor, so it should help those who think in those terms!

I can't see how to scan and post via photobucket but you can find it here:
http://www.arvelgentry.com/techs/A%20Review%20of%20Modern%20Sail%20Theory.pdf

The bits you want are sections 2 to5

I think you will find it deals pretty clearly with most of the issues that have been raised in this thread. But as Gentry himself said in an interview:


Arvel notes: "Aerodynamics is a difficult subject, and all attempts to simplify it for the average person leads to wrong interpretations. The facts are that lift comes about because air has viscosity, which leads to the starting vortex. This is followed by the formation of a circulation field about the airfoil necessary to meet one of Helmoltz's theorems of vortex motion. Then the Kutta condition is satisfied at the trailing edge, and bingo -- we have lift. These principles, together with knowledge of boundary layer theory, lead to a correct understanding of the interaction between the jib and the mainsail."
Easy for him to say, right?
Gentry puts is as "You can't just sit there and stare at your navel and come up with conclusions."

(http://www.arvelgentry.com/techs/A%20Review%20of%20Modern%20Sail%20Theory.pdf)

mm43
21st Oct 2012, 06:52
You can't just sit there and stare at your navel and come up with conclusions.From where I'm perched; aft of the jib and abeam and to the lee of a full mainsail, the conclusion is obvious.:ok:

Owain Glyndwr
21st Oct 2012, 07:10
mm43

From where I'm perched; aft of the jib and abeam and to the lee of a full mainsail, the conclusion is obvious

Wish I could be with you, but I'd prefer to be further aft and on the windward side with a tiller in my hand ;)

ft
21st Oct 2012, 11:37
One of the things which continues to throw a spanner in the works each time this subject is broached is the fact that people look at Bernoulli's theorem but only see the equation and forget to look at the complete definition, especially the limits to its applicability.

It applies to a steady flow of an incompressible, inviscid fluid.

If you have compressibility, if you have viscosity, if it's not steady - then Bernoulli does not apply, and will not yield 100% correct results. The error will depend on how much your application deviates from the defined required conditions.

Often, the errors can be ignored for all practical purposes. Outside of the boundary layer (but you have to agree on a definition of the boundary layer - leaving the search for the commonly accepted definition as an exercise for the interested reader) and at low airspeeds where compressibility isn't much of a factor, it'll generally be good enough.

That article seems to want to throw Bernoulli out the window as it doesn't apply in the parts of the flow where viscosity is significant. In my native language, we have a saying about kicking in open doors. That would apply, I think. Noone knowledgeable, especially not mr. Bernoulli himself, has ever claimed that Bernoulli's theorem is without limitations.

Lyman
21st Oct 2012, 16:45
Hi ft...

"It applies to a steady flow of an incompressible, inviscid fluid."

I think that is the point I attempted to make, inelegantly.

The authors of the paper in question make an elegant case for a new way to teach "Introduction to Lift". The math and the relationships among the variables are clear, and persuasive.

The "sacred" maths and several independent theorems are available on Wikipedia, I checked. I have never understood the apparent need to make lift complex, and pay homage to a Swiss hydrologist.

Any new approach will make "waves" and a vortex or three. Toes will be compressed in its forthcoming popularity...

As to the lack of facts in this post, I will incorporate the paper here, by reference.

italia458
21st Oct 2012, 20:52
Lyman...

I have never understood the apparent need to make lift complex, and pay homage to a Swiss hydrologist.

Lift is complex! And Bernoulli wasn't a hydrologist, he was a physicist and mathematician. Daniel Bernoulli (http://en.wikipedia.org/wiki/Daniel_Bernoulli) versus a hydrologist (http://ga.water.usgs.gov/edu/hydrology.html).

I think the quote that Owain posted needs to be emphasized more. Until you accept this, I see no point in continuing to try to understand the complexities of lift. A scientist or physicist or aerodynamicist all have one thing in common... they all are studying nature and discovering the way it is. If it turns out the way nature is is simple, then that's the way it is. If they find out that nature is complex, then that's the way it is! Lift happens to be one of those complex things.

"Aerodynamics is a difficult subject, and all attempts to simplify it for the average person leads to wrong interpretations." - Arvel

That is fundamentally the point I try to make, the wing compresses air...

When studying lift at low speeds, which is where you always start, it can be said that air is incompressible at speeds below Mach 0.3. That satisfies one of the conditions of Bernoulli. Outside of the boundary layer air can be considered inviscid, another condition for Bernoulli.

Henra...

Yes, expansion means increasing pressure. But increasing can mean increasing back to free stream static pressure. No discrepancy to the 'closed' system here.

Yup... I made an error with my description. What you say makes sense.

Owain Glyndwr...

I disagree. The energy added is used to overcome a side effect of lift generation – drag.

There is abundant evidence from wake survey experiments that total pressure is not constant behind a wing producing lift, so in broad terms they are right, although not because energy is added – rather it is subtracted. However, it is also true that this loss of total pressure is confined to a small area just behind the TE – in the wing wake in fact.

Would it be correct to say that the flow outside the boundary layer does not have energy added or subtracted (and it's incompressible and inviscid) so Bernoulli applies to it? Is the boundary layer then responsible for the induced and parasitic drag?

I'm trying to get an understanding of the energy of the system - how energy is added or subtracted to the air by an airfoil passing through. It makes sense that energy has to be added to overcome drag. Regarding the loss of total pressure behind the TE, where does that energy go?

Owain Glyndwr
21st Oct 2012, 21:50
Italia458

Would it be correct to say that the flow outside the boundary layer does not have energy added or subtracted (and it's incompressible and inviscid) so Bernoulli applies to it?Yes, Bernouilli's equation is, for all practical purposes, valid outside the boundary layer - but note ft's comment that strictly speaking you have to define the limits of the boundary layer.That can be done in several ways but the fine differences only matter to aerodynamic pedants (I'm not one I hope) :8

Is the boundary layer then responsible for the induced and parasitic drag? The boundary layer is not "responsible" for drag in any direct sense of course. It is the viscous forces associated with the velocity shear inside the boundary layer that produce skin friction drag.
If you are looking at a 2D wing then there will be no induced drag in the classic sense since you have an infinite aspect ratio. For sensible finite wings then there will be drag due to lift - for a wing in inviscid flow and with an elliptical loading that will be Prandtl's classic CL^2/(Pi*A.Ratio). For non-elliptic loading and the effect of fuselage Europeans usually put an induced drag factor 'k' in front of that. In the USA it is more common to use the Oswald efficiency e = 1/k. This bit of drag due to lift has nothing to do with the boundary layer.

In real life viscosity and the effect of pressure gradients on the upper surface mean that as AoA (CL) is increased the boundary layer flow will start to separate and the drag will increase. In practical measurements this shows up as an increase in 'k'.

So boundary layer flow is involved in both skin friction drag and part of the induced drag. If there are any separations around at zero lift it can also be involved in the pressure drag.

Regarding the loss of total pressure behind the TE, where does that energy go? Never really thought about it deeply, but since the energy loss comes from frictional forces why doesn't it (eventually) show up where friction effects always do show up - heat! [That isn't taken into account by Bernouilli either].

Going to be offline next week, but parting thought - if you want a quantitative explanation of lift generation then you have no choice but the circulation explanation L = rho * circulation* airspeed, but if you are happy with a qualitative explanation you can opt for the Newtonian L = mass flow rate * downwash. Neither of those two parameters can be defined numerically (see below) so it doesn't satisfy me, but if you can live with that well "Chacun a son gout" BTW, Bernouilli's equation doesn't figure in either of those explanations
;)

[We don't know the depth of the region affected by the wing or even its shape; downwash is not uniform over this region, in fact it varies with distance below the wing]

A Squared
23rd Oct 2012, 16:07
That is fundamentally the point I try to make, the wing compresses air,.....

Nope. Wrong again. At the airspeeds encountered in a typical general aviation aircraft, (less than 200 knots) there is no significant compression of the air.

Air is not a liquid Uhh, yeah, for the purposes of understanding the aerodynamics of a low speed airfoil it is in fact, essentially an incompressible fluid. There is no significant change in the volume of the air as it flows around an airfoil at say 150 knots. A change in pressure, but not in volume.

This gets back to my earlier comment. Most of what you "know" is wrong.

It is the wing that moves, not its medium...

A meaningless distinction. What matters is that there is relative motion. Which is moving and which is not is completely dependent on the frame of reference, arbitrary, and completely irrelevant.

The fact that you beleive that this is meaningful is only an illustration of how poorly you understand physics.

HazelNuts39
23rd Oct 2012, 16:16
We don't know the depth of the region affected by the wing or even its shapeWell, at subsonic speeds, it is infinite, in all directions. Any boundary is arbitrary.

Lyman
23rd Oct 2012, 17:05
HazelNuts39

"Well, at subsonic speeds, it is infinite, in all directions. Any boundary is arbitrary."

AA

"Nope. Wrong again. At the airspeeds encountered in a typical general aviation aircraft, (less than 200 knots) there is no significant compression of the air."

"A meaningless distinction. What matters is that there is relative motion. Which is moving and which is not is completely dependent on the frame of reference, arbitrary, and completely irrelevant."

How is it all of a sudden an energy source is not relevant? It is consistent with your view of Physics as squishy, and dependent on your definitions, definitions that involve a suspension of actual Physical Laws.

Any widely held theory that depends entirely on suspension of fundamentals seems to attract hysterics..

You say...

"A meaningless distinction. What matters is that there is relative motion. Which is moving and which is not is completely dependent on the frame of reference, arbitrary, and completely irrelevant" (my bolding, throughout)


Air is the recipient of added energy, not the source. Watch your landscaper blow debris about with a leafblower, you will perhaps see my point.

The source of added energy is not important? Strange viewpoint from an expert.

roulishollandais
23rd Oct 2012, 17:09
strictly speaking you have to define the limits of the boundary layer.That can be done in several ways but the fine differences only matter to aerodynamic pedants (I'm not one I hope
...If mathematicians are aerodynamic pedants, I accept the mockery at the expense of revenge .. :)
rh

henra
23rd Oct 2012, 18:50
Air is the recipient of added energy, not the source. Watch your landscaper blow debris about with a leafblower, you will perhaps see my point.

The source of added energy is not important? Strange viewpoint from an expert.

Although I don't want to get involved in this somewhat personal argument I'm afraid he is right insofar as for the balance inside a certain frame of reference it is absolutely irrelevant which of the two is moving.
Example?
The fact that the surface of mother earth is moving at 1500km/h plus at the equator has no influence on the behaviour on the airfoil either. Aerodynamic behaviour at the North pole will be the same.
You get the point?
And btw. you don't create energy, you don't lose it, you can only change the form in which it is present. And that also depends on the frame of reference.

Lyman
23rd Oct 2012, 21:13
henra

"And btw. you don't create energy, you don't lose it, you can only change the form in which it is present. And that also depends on the frame of reference."

Show me please that I posited energy was "created"?

You make the mistake my esteemed opponent is making. Assumption after assumption. To gather an air mass and shape it such that it displaces and affiliates to the structure of the wing is impossible, in this universe. There is a flow, a sequence, that is not well served by constantly defining terms that allow a reversal of logic.

To the student, it is difficult enough to "fix" a wing, and parlay that into movement, outside the enclosed volume.

If you want to discuss a blown wing, fair enough, but to parse a gaseous volume into boundaries ahead of the entry of the wing is bizarre...

Lyman
23rd Oct 2012, 22:25
I think we are having two distinct discussions. I refer again to the paper linked by Peter Kent. The authors refer to a 'Real Life' wing......

I can summarize in this way. A cambered airfoil produces lift at 0 degrees Angle of attack, AoA as centerline through chord LE/TE.

Remove this definition from the argument. Can a symmetric airfoil produce Lift?
Of course yes. So camber is not a critical precursor of Lift? Of course not.

So if camber is not a requirement, how is it included in the basic description?

These two fellas propose a simple way to understand lift. One that does not require suspension of 'aspects' of gases, and motion.

Owain Glyndwr
27th Oct 2012, 09:23
Hi Hazelnuts,

Well, at subsonic speeds, it is infinite, in all directions. Any boundary is arbitrary.


Sure it is, but you also wrote:
it can be shown for an elliptic lift distribution that the area of the scoop is equal to a circle with a diameter equal to the wing span.


so the area of the affected stream tube is a finite part of an infinite field - doesn't change my view that one cannot quantify mdot ;)

HazelNuts39
27th Oct 2012, 10:31
OG;

Yes, if one arbitrarily assumes ... that all the mass captured in the "virtual scoop" uniformly gets a downwash velocity vv, the power required for this is Pi=0.5*mdot*vv^2. Equating this to the power Pi=Di*V to overcome the induced drag Di, ...

EDIT::
OTOH, if one equally arbitrarily assumes that the uniform downwash velocity is that predicted by Prandtl for the 'lifting line', the area corresponding to the lift momentum mdot*vv is twice as large.

Owain Glyndwr
27th Oct 2012, 12:25
OTOH, if one equally arbitrarily assumes that the uniform downwash velocity is that predicted by Prandtl for the 'lifting line', the area corresponding to the lift momentum mdot*vv is twice as large.

Quite :ok:

A Squared
27th Oct 2012, 19:20
How is it all of a sudden an energy source is not relevant? It is consistent with your view of Physics as squishy, and dependent on your definitions, definitions that involve a suspension of actual Physical Laws.

Any widely held theory that depends entirely on suspension of fundamentals seems to attract hysterics..

Lyman, the laws governing the behavior of energy are completely independent of frame of reference, as long as that frame remains consistent throuought the analysis.

That is fundemental, first day, elememtry physics, yet you want to mock it as "squishy". This ain't "squishy", it's a fact. You can look it up. There's really no point in arguing fundamental, verifiable fact with somone who doesn't have a grasp of them. You really don't have a clue how badly misinformed you really are, do you?

Air is the recipient of added energy, not the source. Watch your landscaper blow debris about with a leafblower, you will perhaps see my point.


Your insitence that there is something very significant and meaninful in the fact that the airfoil is moving through the air as opposed to the air moving over the airfoil would necessarily demand that a completely different set of aerodynamic principles would keep a J3 aloft when maintaining a stationary position over the earth, flying into a 70 knot headwind than those keeping it aloft when it is flying 70 knots over the earthe surface through still air.

Lyman
27th Oct 2012, 20:16
The energy that creates lift comes from the aircraft, not the airmass. You are confusing result with cause. That is the upshot of the paper in question.

Shut the little Franklin engine down? Oh oh. You are so welded to a construct that you cannot imagine a "real-life" wing? For your purpose, donor/recipient is not important. Fine, but that leaves the Aero 101 student to catch up with you.

Like all the other assumptions you make, or remember, you miss the point.

You cannot posit an a/c under power, and then dismiss that power. That is just sloppy work on your part.sloppy. It is power creates the lift; the airmass is a bystander, regardless its relative motion v/v the Earth's surface. Your example is useless.

Why do you ignore airfoil shape? Do you disagree with me, that shape does not play a part in Lift, at its most basic?

I think the motionless wing and "moving air" may have caused the misconceptions that the authors elucidate. Since the air is "moving" it must have energy, right? Since it has energy, it can create its own low pressure area, right?

Having created its own LP, against the Laws of Newton, Lift is created? That is wrong....it does not help to model a wing that produces lift at "Zero Angle of Attack". Magic air, defying Newton, and magic wing, creating Lift from "zero".

Both incorrect, and the source of much confusion, stemming from a sloppy model.

One "assume" after another, and then come two guys ready to provide a workable Newtonian paradigm.......

To build a working model to demonstrate there is no relevance to the energy source that produces identical airflows locally and remotely is impossible. Unless you can confine cubic miles of air and accelerate it as a unit to flow directionally past a fixed airfoil, you need to "imagine" or to "postulate" such a flow. You have my permission to attempt such an experiment.

Likewise, feel free to imagine that air is not viscous, nor possessed of mass.

henra, have you located a quote where I say that energy is "created"?

TURIN
28th Oct 2012, 22:31
I've been dieing to get into this thread but after reading the whole thing from the begining I am convinced it's just a wind up.

What were all those tubes fastened to an aerofoil in a wind tunnel showing me all those years ago. There was definately a pressure difference.

Definitions of angle of attack???

Ye gods. Chord line and airflow, (direction of flight).

Camber has ne effect on lift? What!!!


This is my favourite...

To build a working model to demonstrate there is no relevance to the energy source that produces identical airflows locally and remotely is impossible. Unless you can confine cubic miles of air and accelerate it as a unit to flow directionally past a fixed airfoil, you need to "imagine" or to "postulate" such a flow. You have my permission to attempt such an experiment.

Been there, done it and watched the sunset from 1000ft agl, parked into wind, hands off, going nowhere, dynamic (ridge) lift over the Derbyshire Peak Ditrict (UK).

I do not apologise for not understanding the deep maths quoted but try this, wrap the first inch or so of an A4 piece of paper around a pencil. Blow over the top of the paper, it will rise. Why? Diff pressure? Diverted airflow down somhow pulls the paper up?

Discuss.

I'll be back in a week.

Lyman
29th Oct 2012, 00:44
Hi TURIN

"Camber has no effect on lift? What!!!"

Who said such a thing? Not I.

From A Squared....

"Lyman, the laws governing the behavior of energy are completely independent of frame of reference, as long as that frame remains consistent throuought the analysis."

I think we are talking past each other. A wing flying through Still air produces specific characteristic flow. If you propose to isolate this system and Examine the airflow, then you are correct, a dynamic system can be analysed that way.

The point is, to get identical airflow by anchoring the wing, and accelerating the air mass, You can not possibly replicate the specific airflows produced by the first system. The air is not still. It is not still because no way has been found to accelerate an airmass in perfect uniform fashion, you will get turbulence.

Again, the paper under discussion proposes a linear and sequential process. The supposition is still air, a moving wing, and camberless chord section.

Since the proposal is to replace the standard model with Newton, the model is more basic, and seeks to eliminate camber and false premise from the model.

By false premise they refer to the mistaken, and seemingly taken for granted meme that air creates a low pressure by accelerating. Since they clearly show that the old model relies on conservation of energy, there is no additive energy, and the premise cannot be correct. It is low pressure that creates the acceleration.

TURIN, in your quote above, the evidence is that you had a conclusion before you understood my statement. You expected that what I was saying is that camber does not affect lift, so even though I said no such thing, you chose to conclude that, in astonishment....

What I said is that Camber is not necessary to explain lift, further, it truly does not do so. It can be used to explain the development of a high pressure area. The wing mightily disrupts airflow, yes?

Owain Glyndwr
31st Oct 2012, 11:28
To judge by recent postings one might assume that any debate on lift generation is done and dusted and that the simple (simplistic?) Newtonian explanation is the only game in town. But the only fault, and it was a major killer fault, in the so called “standard” description of lift generation was that the explanation offered for different velocities on upper and lower surfaces was a load of crap. Nobody AFAIK, disputes that the air flows faster over the upper side of a lifting wing than over the lower, and no one with any real understanding of basics has any problem with using Bernouill’s theorem to associate those velocity changes with pressure changes that give lift.
Any half decent scientific explanation should cover all known facts, which in the case of a lifting wing are not only the upper/lower velocity differential but also the generation of upwash ahead of the LE and downwash at and behind the TE. In addition it is well established that if one moves vertically either up or down from the TE the downwash reduces. The Newtonian explanation ignores most of these.
But what if there should be a valid alternative explanation for the velocity differences? The rest of the standard model then comes back into play giving a complete alternative to the Newtonian explanation.
With apologies to both those who think the following is dumbed down and those who think it too complicated, this is an attempt to put the math into plain words. If you are happy with the Newtonian explanation far be it from me to seek to change your mind, better switch threads or have a drink and watch a ballgame on TV, but if your curiosity extends further it might be worth a read.


http://i1081.photobucket.com/albums/j351/OwainGlyndwr/Liftgenerationtheory.jpg


The basic premise is that the flow around a lifting wing has two components (bottom left) – an "irrotational" flow plus a rotational velocity generated by a vortex whose axis lies along the wing quarter chord line. It should, I think, be fairly straightforward to see how such a rotational flow would increase velocity over the upper surface and reduce it over the lower. Note that this is NOT suggesting that any particular air molecule does circles around the wing section. Because the freestream velocity is greater than the rotational velocity the air is always swept past the airfoil on one side or the other – it just takes longer on the bottom.


Where most get stuck is seeing how such a vortex might come about; isn’t it a figment of a mathematician’s imagination? I think not, we have all seen part of this vortex. A picture is worth a lot of words they say, so this is a composite from several sources which I hope might serve as a plain man’s guide to lift generation.
What you need to know is that a vortex cannot suddenly start or stop in midair. It must either go on to infinity or be a closed system. In the case of a finite wing it is the latter. When the wing vortex (technically called the bound vortex) arrives at the wingtip the changed flow conditions cause it to change direction and continue rearwards as a trailing (aka wingtip) vortex. Since for any vortex the velocity increases and the static pressure drops as you move towards the core if the conditions are right any water vapour condenses out and we “see” the vortex as a contrail. These vortices are real!

All well and good perhaps, but how does the vortex system get started?

Moving from six o’clock anticlockwise back to three we start with the theoretical streamlines around a flat plate at incidence. In a perfect fluid the underwing flow can move around the TE and join up with its mates on the upper surface (btw it needs to develop infinite velocity as it does so!). In real life of course the flow separates, but in an attempt to flow round the corner it develops some swirl. This rolls up into what is described in the jargon as a ‘starting vortex’. Once again, this is real and the flow visualisation photograph (taken by Prandtl) shows such a starting vortex (which is in fact generated when the wing first starts to move through the fluid).
So now we have a closed vortex system. Three sides of the rectangle we can see in the right circumstances, the fourth, along the wing is more difficult, but it is there. Its presence explains not only the velocity differential between upper and lower wing surfaces but also the existence of upwash ahead of the LE, downwash behind the TE and the variation of downwash with height above or below the TE. Once you have that of course an application of Bernouilli's theorem gets you to suctions on top and pressure below and Bingo, you have lift.

I hope this is not too confusing, and I certainly hope it does not read as patronising, but having consulted an eminent PPRuner we thought the terms of the discussion might be widened – as someone said recently (I think the post has been deleted) – it is a technical forum after all.

John Farley
31st Oct 2012, 15:12
Thanks Owen for bringing a touch of reality and truth to what I feel has been generally one of the most incorrect threads ever on the subject.

Brian Abraham
1st Nov 2012, 00:18
(I think the post has been deleted) – it is a technical forum after all. Owain, that was me. I became so frustrated with the nonsense being posted I deleted all my posts. An individual claims he wants to learn, but is dismissive of scientific facts presented, seemed little point in continuing. I doubt he will accept your profferings, excellent though they be. Keep up the good work if you have the stamina and fortitude.

Owain Glyndwr
1st Nov 2012, 07:45
An individual claims he wants to learn, but is dismissive of scientific facts presented, seemed little point in continuing. I doubt he will accept your profferings

Brian, I am prepared for that; there is a mental autoignore list. My intended "audience" was others who might be tempted to believe the rubbish being posted.

john_tullamarine
1st Nov 2012, 09:58
If I may be permitted the luxury of indulging in a little heresy ...

One of the benefits in having a variety of views expressed -expert (and for those who don't know who OG is .. he is, most assuredly, an expert in these matters) ranging through to technically naive or even incompetent - is that the various levels of technical competence represented within our group are provided with the opportunity to test and challenge their particular knowledge sets.

This process, if approached in a mature way, results in learning and general improvement of knowledge.

If Tech Log achieves nothing else during the life of PPRuNe, such education as might be facilitated ... will be its legacy.

Lyman
1st Nov 2012, 13:52
Howdy. Much of the "rubbish" I brought up was from the paper written by the two boffins. I will own my own, but in dismissing mine, without addressing the points from the Newton screed, well....baby/bathwater?

Specifically, Newton's second Law defeating any added energy from the airmass, per "bernoulli"? Shape of airfoil as irrelevant to lift, save at Stall?*

Owain, if your fear is that some innocents may be harmed by rubbish, could you address the scientists' 'rubbish' specifically?

Re the condensation above the DC10 wings, and the low pressure that created it.

My question is a general one. From whence comes the force supporting that monstrous mass in the air? Beneath the wing, as pressure, or above the wing, as "pull"?

Thanks for your patience.....



* "We should point out that the shape of the wing does affect the stall characteristics and efficiency of the wing, but it is not the primary factor in determining its lift."

Owain Glyndwr
1st Nov 2012, 16:22
Owain, if your fear is that some innocents may be harmed by rubbish, could you address the scientists' 'rubbish' specifically? I didn't say that the Anderson/Eberhardt paper was rubbish. I think it simplifies things a little too much and it can be misleading. For example this business that Bernouilli's equation cannot be applied to the flow around a wing (or indeed any surface where there is an energy transfer from body to air). As they say, Bernouilli's equation applies in a constant energy situation, and strictly applies along a streamline. But the energy variations are confined to areas inside the boundary layer where the flow is chaotic and there are no streamlines. However, there is no transverse static pressure gradient across the boundary layer, so the pressure at any point on the actual wing surface is the same as that immediately above that point on the streamline at the edge of the boundary layer where Bernouilli can be applied.

But if I criticise some points I should also say where I agree. For example, they are correct when they say:
The streamlines are bent by a lowering of the pressure. This is why the air is bent by the top of the wing and why the pressure above the wing is lowered. This lowered pressure decrease with distance above the wing but is the basis of the lift on a wing


Put another way [Holger Babinski "How do wings work?" ww.iop.org/journals/physed



In other words, if a streamline is curved, there must be a pressure gradient across the streamline, with the pressure increasing in the direction away from the centre of curvature.
This relationship (derived mathematically in the appendix) between pressure fields and flow
curvature is very useful for the understanding of fluid dynamics (although it doesn’t have a name). Together with Bernoulli’s equation, it describes the relationship between the pressure field and the flow velocity field.
This may be at right angles to the direction of acceleration most think of in connection with wing flow, but it is correct.

Again, my diagram was an attempt to explain circulation theory. Their paper says:


Yet another common description of lift is that of circulation theory. Here the air is seen to rotate around the wing. This is sometimes used to explain the acceleration of the air over the top to the wing. There is a great deal of jargon, such as "starting vortex" and "bound vortices", associated with this description. Circulation theory is a mathematical abstraction useful and accurate for aerodynamic calculations. ....... Circulation is a model
developed for large aircraft ....... and:

Although circulation theory can be used for accurate calculations of lift, it does not give a simple, intuitive description of the lift on the wing

I was trying to give an intuitive description of this way of looking at the problem - a theory they accept as accurate btw.

My question is a general one. From whence comes the force supporting that monstrous mass in the air? Beneath the wing, as pressure, or above the wing, as "pull"?You know the answer to that! Bit of each but mostly from above and most of that from suctions in the first 30% (ish) of the chord.

Lyman
1st Nov 2012, 16:39
Owain...

Thank you for all that, your patience is notable, and I appreciate it.

I would like to pose some questions, of a very basic nature, in the interest of establishing a foundation.

1 How accurate is it to use the term, "flow" when teaching lift? Would a better term be "displacement"? The salient vectors are Normal to the streamlines, not horizontal, yes?

2. So are we saying that the air in proximity to the wing is accelerated instantly to say, 250 knots? Or is it forced to inhabit somewhat less volume than prior to entry of the wing? If it accelerated, would there be enough energy to accomplish lift?

3. Last, I don't believe in suction, because I was taught not to. Differential, yes.
If you do, could I describe suction as:

The inability of the airmass to form a "void"?

Owain Glyndwr
1st Nov 2012, 16:48
I would like to pose some questions, of a very basic nature, in the interest of establishing a foundation.

1 How accurate is it to use the term, "flow" when teaching lift? Would a better term be "displacement"? The salient vectors are Normal to the streamlines, not horizontal, yes?

2. So are we saying that the air in proximity to the wing is accelerated instantly to say, 250 knots? Or is it forced to inhabit somewhat less volume than prior to entry of the wing? If it accelerated, would there be enough energy to accomplish lift?

3. Last, I don't believe in suction, because I was taught not to. Differential, yes.
If you do, could I describe suction as:

The inability of the airmass to form a "void"?

Sorry Lyman, but when you ask "questions" like that my notable patience runs out.

Lyman
1st Nov 2012, 16:55
Those are earnest and sincere questions, do you not understand them? Do you fear them? Are they rubbish?

Is the discussion ended?

awblain
1st Nov 2012, 21:15
I suggest that you could also call `circulation theory' a simple piece of legislation by Mr Newton.

The flowing air lifts up the wing, and the wing pushes down the flowing air.

Behind the aircraft, air is now descending, to a good first order, to the tune of `lift = rate of change of vertical momentum in the airflow', while behind the aircraft the air is also made to follow along, to a first order, to the tune of `drag = rate of change of horizontal momentum in the airflow'.

roulishollandais
2nd Nov 2012, 16:56
@Owain Glyndwr,

Thank you for the excellent Arvel E. Gentry document. That can be taught to begin. The most of the false theories are well explained. In his document "streamlines" may be considered as a good pedagogic tool.

My question about "streamline" is still a little deeper : where do you put the turbulence in the unhomogeneous "streamline" ? David Ruelle and Floris Taken (Institut des Hautes Etudes Scientifiques, Bures sur Yvette (France)) showed (1) that turbulence in his phases space has an strange attractor which is of length infinite in the area, and then it is a fractale.
Jerry P. Gollub and Harry L. Swinney showed by experimenting on the Couette-Taylor (2) flux that Lev D. Landau's theory of turbulence was wrong, and they discovered only very few transitions in increasing the speed, and chaos appeared very quickly.

roulishollandais

(1) "On the nature of turbulence" Communications in Mathematical Physics, n°23, 1971,pp.155-183
(2) "Onset of turbulence in a Rotating Fluid" Physical Review Letters n°35, 1975, p.927

Owain Glyndwr
2nd Nov 2012, 18:35
@roulishollandais

I'm not sure I know how to reply to your question roulis. Could you expand a little what you mean by "put the turbulence in the unhomogeneous "streamline" please.

If I simply said that I see the flow inside the boundary layer as essentially wholly turbulent (since transition on practical wings is never aft of the slat TE) and that inside this turbulence the flow is chaotic (strange attractor/fractal or whatever description) so that the concept of streamlines is irrelevant, would that meet your needs? Streamline flow then is limited to that region outside the boundary layer (however you define that)

roulishollandais
12th Nov 2012, 17:35
@Owain Glyndwr,

outside the boundary layer

The problem with streamlines exists inside and outside of the boundary layer.
You have chaos everywhere turbulence exists...

Site Web pour cette image (http://www.princeton.edu/artofscience/gallery2006/view.php%3Fid=32.html)

Turbulence is chaotic by definition, ...

princeton.edu

http://www.princeton.edu/artofscience/gallery2006/images/32.jpg

FlightPathOBN
12th Nov 2012, 18:40
When the wing vortex (technically called the bound vortex) arrives at the wingtip the changed flow conditions cause it to change direction and continue rearwards as a trailing (aka wingtip) vortex. Since for any vortex the velocity increases and the static pressure drops as you move towards the core if the conditions are right any water vapour condenses out and we “see” the vortex as a contrail. These vortices are real!

The only part of that statement that is correct is "these vortices are real"

The illustrations that are provided with the entire post are oversimplified and misapplied to wake turbulence...
http://i1081.photobucket.com/albums/j351/OwainGlyndwr/Liftgenerationtheory.jpg

This is likely the result of a reliance on CFD models, and other nonsense that has prevailed for many years, but is now thankfully being addressed. These illustrations show a wing section in a wind tunnel, with simulated winds. Smoke or other methods are used to detect the airflow around the wing section, or even a wave tank with colored water.

Notice that the airflow in the section is not deflected? How is lift generated of airflow is not deflected?

Rollup of the vortices occurs at the location of the wing where the laminar flow over the top of the wing disconnects from the top of the wing surface, NOT at the wingtips. (notice on your illustration that the condensation trail originates from the outward edge of the flap)
Here is a better example...757 (note how wing/outboard flap config affects vortex gen? just wait until you see the vortex behind a 787....
http://blog.aopa.org/asfblog/wp-content/uploads/2008/11/vortices.jpg

The rollup...

http://www.iasa.com.au/folders/Safety_Issues/RiskManagement/waketurber_files/c-17turb.jpg

What has been oversimplified is the illustrations noted. There is no accounting for the weight and varied surfaces of the aircraft/wing, and in general, the overall issue that air can be compressed, while water cannot.

It is relatively easy to see the illustrations that have used CFD, such as those you have shown, but as we are finding out...cannot explain wake vortex creation.

Just look at winglets touted as reducing wake as a great example.

http://operationsbasednavigation.com/wordpress/wp-content/uploads/2011/06/WV3.jpg

Lyman
12th Nov 2012, 19:17
Hi FPOBN....

Do you have a permission slip to inject compression and viscosity?

Cheers

FlightPathOBN
12th Nov 2012, 20:36
plausible deniability...:rolleyes:

Owain Glyndwr
13th Nov 2012, 14:43
roulishollandais
Nice pictures!
:ok:
But they are theoretical evaluations of the flow in a channel and the flow is subject to three (open channel) or four (closed rectangular channel) boundary layers. With fully developed boundary layers there is no real freestream conditions.
I would be surprised if the results shown resulted from anything more complex than application of a single initial condition. But we know that the output from strange attractors is extremely sensitive to initial conditions (butterfly wing flap to tornado). In real life the boundary layer will be subjected to an infinitely variable and random set of initial conditions as it passes over rivet heads, steps, gaps etc. So I suspect that the chaos in the boundary layer flow will be a lot more complicated than that shown in your photos.
]If we do consider freestream of course then there will always be some level of turbulence, but we must consider how the scale of the turbulence relates to the scale of the object passing through. [Scale here means the magnitude of a characteristic physical dimensional quantity such as wavelength]
Nobody would expect to see a neat set of streamlines surrounding an aircraft flying through a patch of turbulence, and the characteristic length used for load prediction is 2500 ft. I don’t know what the characteristic length would be for a hand launched model glider on a calm day, but I bet it isn’t as high as 762 meters!
If the air is calm (small scale turbulence) then although you would be correct in saying that there must be some turbulence the effect of this at distances greater than the boundary layer height away from the solid surface will be small and for all practical purposes we may assume streamline flow.
The picture below (taken by Prandtl) shows what I mean

http://i1081.photobucket.com/albums/j351/OwainGlyndwr/streamlines.jpg

@FlightPathOBN

Ah well, PJ2 told me I would get into difficulties with tip vortices ...
When the wing vortex (technically called the bound vortex) arrives at the wingtip the changed flow conditions cause it to change direction and continue rearwards as a trailing (aka wingtip) vortex. Since for any vortex the velocity increases and the static pressure drops as you move towards the core if the conditions are right any water vapour condenses out and we “see” the vortex as a contrail. These vortices are real! The only part of that statement that is correct is "these vortices are real" Let me take that bit by bit....
When the wing vortex (technically called the bound vortex) arrives at the wingtip the changed flow conditions cause it to change direction and continue rearwards as a trailing (aka wingtip) vortex.

To use someone else’s words (taken from Wikipedia, my emphasis)

Three-dimensional lift and the occurrence of wingtip vortices can be approached with the concept of horseshoe vortex (http://en.wikipedia.org/wiki/Horseshoe_vortex) and described accurately with the Lanchester–Prandtl theory (http://en.wikipedia.org/wiki/Lifting-line_theory). In this view, the trailing vortex is a continuation of the wing-bound vortex inherent to the lift generation


I stick with my statement


Since for any vortex the velocity increases and the static pressure drops as you move towards the core if the conditions are right any water vapour condenses out and we “see” the vortex as a contrail

Sorry, but you are just plain wrong when you say this isn’t true – again taking words from Wikpedia as I can’t be bothered to look up more erudite references



Depending on ambient atmospheric humidity as well as the geometry and wing loading of aircraft, water may condense or freeze in the core of the vortices, rendering them visible.
The core of a vortex in air is sometimes visible because of a plume of water vapor caused by condensation (http://en.wikipedia.org/wiki/Condensation) in the low pressure and low temperature of the core; the spout of a tornado is a classic example



The illustrations that are provided with the entire post are oversimplified and misapplied to wake turbulence...Well I prefaced my explanation with :
[quote]With apologies to both those who think the following is dumbed down and those who think it too complicated, this is an attempt to put the math into plain words.
And I never once mentioned wake turbulence – I was attempting to explain the circulation theory of lift generation on the wing.

This is likely the result of a reliance on CFD models, and other nonsense that has prevailed for many years, but is now thankfully being addressed. These illustrations show a wing section in a wind tunnel, with simulated winds. Smoke or other methods are used to detect the airflow around the wing section, or even a wave tank with colored water.


Let me say that I am an old fashioned aerodynamicist – I have never in my life used or calculated using CFD. Similarly, the illustrations I used predated extensive use of CFD by many years – Shevell (1983), Gentry (1981) and even Prandtl (1908 I think). So I think your argument that the explanation results from over reliance on CFD is misplaced – and incidentally I don’t think you would find many modern aerodynamicists who would agree with you that CFD is “nonsense”!


Notice that the airflow in the section is not deflected? How is lift generated of airflow is not deflected?


Good spot! But the reason the airflow is not deflected is that this was a one of a sequence of photographs (taken by Prandtl) and chosen by me to illustrate the starting vortex formation. At that point in time the starting vortex had barely left the wing TE and the opposing wing circulation had not been generated. No circulation/no lift and no air deflection (downwash).
But a very short time later in the sequence the starting vortex has moved away, there is circulation around the wing and there is downwash, as in the picture above.

Rollup of the vortices occurs at the location of the wing where the laminar flow over the top of the wing disconnects from the top of the wing surface, NOT at the wingtips. (notice on your illustration that the condensation trail originates from the outward edge of the flap)


I think you must have a completely different conception of laminar flow to that held by most people working in the field. There is no laminar flow over pretty well the whole of the wing top surface (or over the whole aircraft for that matter. The boundary layers are turbulent as shown by roulishollandais photos above. I think what you mean is simple attached flow?


What has been oversimplified is the illustrations noted. There is no accounting for the weight and varied surfaces of the aircraft/wing, and in general, the overall issue that air can be compressed, while water cannot.


As I have said, I was not trying to do anything other than explain how lift is generated. Weight, control surfaces, compressibility are all extraneous factors with no relevance to a simple explanation of lift although they can obviously affect wake turbulence which I understand to be a preoccupation.

It is relatively easy to see the illustrations that have used CFD, such as those you have shown, but as we are finding out...cannot explain wake vortex creation.

As I said, I tried to keep it simple, but since you have brought it up, the explanation for the strong vortices off the flap tip is that the lift/unit span is given by rho*airspeed*Circulation. Over the part of the wing covered by flaps the lift/unit span is much greater than that on the unflapped bit. Since rho and airspeed are the same that means that the circulation is much bigger over the flapped part so that when the flap stops there is an abrupt change in circulation - and circulation is simply the strength of the vortex. Vorticity cannot just start or stop in midair and since the outer part of the wing cannot support it this high level of flap vorticity becomes another trailing vortex. Rather like a wing within a wing perhaps?
The 757 picture you posted is very interesting as it clearly shows the vortex ‘bursting’ some way behind the wing. As AoA increases this bursting point gets closer and closer to the TE.

Mr Optimistic
13th Nov 2012, 15:48
This all started with a 23yr old OP who stated 'Pardon me, this isn't a very technical ques.'

Think he got his monies worth but is he any the wiser ?:)

Lyman
13th Nov 2012, 16:03
I have never understood the apparent need of some people to instantly complicate what it is they do, or believe in. It is beyond me, and is probably a topic for a social science website.

I take a look at the wake of the C-17, and note an immense trough of disturbed air beneath and behind the aircraft. It is notable in its lack of visible moisture, evidently a high pressure zone, perhaps even heated.

The metallic and composite mass that is maintaining its altitude at the expense of an intense disturbance in the airmass remains aloft how? Kutta? Bernoulli? Glyndwr?

I make it as the result of a massive deflection of mass, period.

Can we discuss this without instantly devolving into a willy waving contest to impress? Attack? Defend? Promulgate? Dismiss? Promote?

edit... "Ah well, PJ2 told me I would get into difficulties with tip vortices ..."

So, Owain, maybe not so soon with the wing tip? Maybe the wing itself, and the paper on Newton? Simple concepts for simple people?

Luddite pilot......

Owain Glyndwr
13th Nov 2012, 17:31
As I said at the beginning:

If you are happy with the Newtonian explanation far be it from me to seek to change your mind, better switch threads or have a drink and watch a ballgame on TV,

roulishollandais
13th Nov 2012, 17:33
@Owain Glyndwr, FlightPathOBN,

I don't have much time now, but immediately Thank you for your contribution.

I liked that pictures only to show turbulence, and to help to forget the post #1 shematic... But yes it is just in a wind tunnel, and Iself often am saying it is a dangerous limitation to explain lift, how aircrafts fly, aso.

This all started with a 23yr old OP who stated 'Pardon me, this isn't a very technical ques.'


Sorry, Lyman !

Mr Optimistic
13th Nov 2012, 21:02
From the op's perspective it is perhaps worth stepping back to observe that lift as conjectured is that net force which acts in a direction opposite to that of gravity and that arises very simply from the net force over the wing surface. The net force is the sum/integral of the pressure over the surface. When a mass of air is deflected it takes force to do that and that force has to be transmitted as a pressure distribution in the flow. Aerodynamics attempts to reconcile the equations which arbitrate between the fluid and solid elements but newton physics is all you need.

FlightPathOBN
14th Nov 2012, 00:44
on a lighter note...Newton rules and Bernoulli drools...



just let that sink in....

I make it as the result of a massive deflection of mass, period.


agreed, if the deflection of the combined compressive air mass did not equal the weight of the aircraft...you would have?
for every force the is an equally reactive force...

after all, it is called an air PLANE...not an air LIFT



:D

Owain Glyndwr
14th Nov 2012, 09:13
In the conclusions of the Anderson/Eberhardt paper that Lyman cites so often the authors say:
Although circulation theory can be used for accurate calculations of lift, it does not give a simple, intuitive description of the lift on the wing.
I admit I thought (hoped) that professional pilots might be interested in an accurate description of how lift is generated and smart enough to understand it if presented in non-mathematical language. Seems like both expectations were optimistic.

So Brian Abrahams was right - it is a waste of time and I will follow his lead by leaving you to wallow happily.

But allow me a parting question. Would some advocate of the Newtonian explanation tell us how it explains the presence of those strong vortices at the outer ends of flaps or wings? Looks to me as if there is as much air moving up in those rotational flows as there is moving down. But Newton says all the air is moving downwards doesn't it? [BTW, the explanation should not involve drooling Bernouilli or pressures above the wingtips being lower than the pressure on the lower surface]

Eppur si muove

FlexibleResponse
14th Nov 2012, 14:12
A simple intuitive description of lift?

When an air vehicle slides through the air it displaces downwards a mass of air that can be seen as downwash.

When the mass of air being depressed equals the mass of the air vehicle level flight is possible.

When the mass of air being depressed is less than the mass of the vehicle, then the vehicle descends and when the mass of air exceeds the mass of the vehicle, the opposite occurs.

All the rest of the observables are symptomatic of the process of lift production and are for the Aerodynamicists to measure and worry over so that they may optimize design efficiencies for particular applications (and to confuse pilots!).

Lyman
14th Nov 2012, 14:44
That is a good description. The unfortunate and unnecessary "battle" is joined when any two blokes start to 'defend'.

The paper Owain references is a case in point. Two highly educated and experienced scientists are proposing a better way to TEACH lift. They suggest a return to an emphasis on Newton.

And why not, they list several demonstrably false premises in the history of teaching Bernoulli.....

I won't pretend to understand why Bernoulli is preferred by a group of highly skilled and smart people, except to say that Bernoulli is well understood and along with circulation theory, satisfies a desire to unpack the complexities....

If Owain cannot explain a vortex, I won't even try. But I would start with Newton, Mass, energy...... And viscosity....

FlightPathOBN
14th Nov 2012, 15:04
Would some advocate of the Newtonian explanation tell us how it explains the presence of those strong vortices at the outer ends of flaps or wings?

The aircraft is planing through the air, the weight of the aircraft, with angle of attack of the wings, compresses the airmass to provide level flight. (notice as the aircraft fuel load lightens, the angle of attack to fly level is reduced.)
The wing section is constantly changing, with the leading edge sweep angle, thickness, and width. The combination of angle upward of the bottom of the wing, and the component airflow direction over the top, causes the rollup, while the air beyond the wing tip will rollover the compressed section of air created by the aircraft passing through....

http://ars.els-cdn.com/content/image/1-s2.0-S0376042110000576-gr2.jpg

A surface vessel wake does not compress the water, and the vessel moving through, does not create a wake that sinks into the water

The aircraft wake is compressed, and with the combination of the 2 counter-rotating vortices, will sink to a certain altitude until crow instability occurs.
http://ars.els-cdn.com/content/image/1-s2.0-S0376042110000576-gr4.jpg

http://ars.els-cdn.com/content/image/1-s2.0-S0376042110000576-gr3.jpg

Lyman
14th Nov 2012, 16:17
From the paper

Power and lift
In aerodynamics, the subject of power requirements is seldom considered. Some introductory textbooks do not even have power listed in the index. In aeronautics this discussion would be about drag, which is a retarding force, the effect of which is proportional to the speed of the airplane. Unfortunately, drag is difficult to derive and is usually presented without derivation.

Which is a shame, but joins some other problems which make the standard model problematic.

One if which, imo, is the immediate suggestion that it is tha air that moves, rather than the wing, which although sustainable logically (albeit 'visually'), adds confusion to the model when presented as introduction...

Because that would suggest the energy is bound in the airmass, rather than in the propulsors. It is reasonable to assume, if the airmass is presented as the energetic mass, that "flow" is present. It is not. Not initially, and that is what the authors isolate as damning of the model. They pointedly state "the acceleration is unexplained".

The air is compressed, and then accelerated downward aft of the trailing edge of the wing. Vortices and chaotic turbulence are artifacts of the wings development of "lift". Which brings up perhaps my most serious criticism, that somehow the aircraft is "pulled up by suction". That to me is ludicrous, as low pressure cannot be created in this instance without compressing (pressurizing) airmass somewhere close by. In this case, it is critical to look to the work as "push" NOT "pull".

It is not incorrect to say that "lift" is developed not by flow, but by preventing it.
Preventing flow into low pressure zones is what wings do, and what makes them useful.

HazelNuts39
14th Nov 2012, 16:44
Which brings up perhaps my most serious criticism, that somehow the aircraft is "pulled up by suction". The wind blows from high pressure to low pressure. Viewed from the high pressure area, it looks like being pushed outwards. In the center of a hurricane, it looks more like suction. It all depends on your point of view.

Lyman
14th Nov 2012, 16:54
Hi HN

Point of View without question. But my comment is framed from the Bernoulli position, that locale is all important, to the exclusion of logic, and entertaining assumptions that require the suspension of physics.....

Inviscid, ideal liquid, "virtual scoop", Accelerated airmass, etc....

I thought the global view was reserved for the Physics shair at Oxford, not Bernoulli?

Am I winning you over?

roulishollandais
14th Nov 2012, 17:16
1. ...take care to that mistake (sleep better !)
Drag and Lift are not connected to gravity direction, but to airspeed direction.

Weight and Power come in balance (or unbalance) with Drag and Lift.

2. For these who find the discussion too difficult, stay with Gentry

3. For our PPRuNe's Flight Gods, perhaps they don't understand fractals, equations, aso. but with their brain and body they understand Lift, Drag, Weight, Power, and evolutions in the air better than all of us.

4. Lift and Drag and viscosity are some of the most difficult things to understand. Our best models are still far away from reality... don't desesperate !

FlightPathOBN
14th Nov 2012, 17:33
http://www.ae.illinois.edu/m-selig/ads/afplots/b737a.gif

http://www.ae.illinois.edu/m-selig/ads/afplots/b737d.gif

Airfoil database.... UIUC Airfoil Data Site (http://www.ae.illinois.edu/m-selig/ads/coord_database.html)

Lyman
14th Nov 2012, 17:40
Bonjour roulis


A theoretical for you. As to streamlines, it is accepted that no matter may pass through them, so as to wing area ( Span x mean Chord). Do these phenomena imcrease the effective area of the wing? We see that the two streamlines, when reconnected, prevent for a short distance any alteration in flow, yes? It is Kutta, at the other end?

Also, this.....

A wind tunnel is created that separates two areas, one, the lower, is air, the upper (above the wing) is completely evacuated, though the pressure is maintained at constant level throughout the tunnel, (bear with me).

The wing is in constant contact with the lower (moving) airmass, there is no mass above the leading edge. Is there "lift"?

Owain Glyndwr
14th Nov 2012, 18:32
@FlightpathOBN

Nice pictures, but I asked for an explanation of how the vortices developed using Newtonian arguments.

The aircraft is planing through the air, the weight of the aircraft, with angle of attack of the wings, compresses the airmass to provide level flight. (notice as the aircraft fuel load lightens, the angle of attack to fly level is reducedSo airspeed doesn't come into it?

The A&E paper says:

The key points are that the lift of the wing is proportional to mdot & vv and the induced power is proportional to Lvv. Also, mdot is proportional to the wing's area and speed, and the air density, while vv is proportional to the wing's speed and angle of attack.


That is mdot (mass flow rate) not air mass, and there is no mention of any air mass being compressed. When we get to details that A&E paper again says:




Thus, a Cessna 172 at cruise is diverting about five times its own weight in air per second to produce lift. A 250-ton jumbo jet in cruise is diverting about its own weight per second. How do you explain that, and why isn't the Cessna a danger to the jumbo not the other way round?

The wing section is constantly changing, with the leading edge sweep angle, thickness, and width. So what about an unswept wing, constant chord and t/c? Doesn't that have tip vortices?



The combination of angle upward of the bottom of the wing, and the component airflow direction over the top, causes the rollup,
Why should the angle upward of the bottom of the wing have any effect on the rotational velocity? What causes the "component airflow direction over the top"? Doesn't a change of direction (from freestream) imply some sort of force (Newton's first law)? So where does this force come from in Newtonian logic?

FlightPathOBN
14th Nov 2012, 19:54
Owain,

Will have to take some time to look through much more of the thread. So many issues have been brought up, and by so many people, it is difficult to see which question(s) were answered, and who asked...

and nothing has tip vortices...

Owain Glyndwr
14th Nov 2012, 20:51
Flightpath
and nothing has tip vortices...Sorry, I thought that was common usage - trailing vortices originating at the tips of a wing with no flap or control surface deflection be OK?

BTW, I've just noticed that on your bottom picture showing how the wing vorticity behind the aircraft rolls up into two concentrated vortices, there are symbols +/- capital Gamma. Capital Gamma (sorry I can't find a way of inserting the Greek letter) is the standard symbol for circulation, so your picture is actually confirming that wake turbulence is proportional to circulation (and wing lift) ;)

awblain
14th Nov 2012, 21:37
How does Newton account for vortices?

Air that's flowed over the wings is going down. Air that's closer to the centerline is going down more, air that's further from the centreline is going down less. Hence shear, which is skating perilously close to rotation.

Owain Glyndwr
14th Nov 2012, 21:50
Air that's flowed over the wings is going down. Air that's closer to the centerline is going down more, air that's further from the centreline is going down less.

Just take a look at Flightpath's pictures - especially the one immediately under "The aircraft wake is compressed, and with the combination of the 2 counter-rotating vortices"

That little symbol 'w' is downwash and the picture shows that the air closer to the centreline is going down a lot less than that near the tip. So I'm afraid your argument is not supported by the facts.

TheChitterneFlyer
14th Nov 2012, 21:59
I'll bet 'extricate' wishes he'd never started this!http://images.ibsrv.net/ibsrv/res/src:www.pprune.org/get/images/smilies/puppy_dog_eyes.gif
I'd like to bet that everyone else wishes that he hadn't started this!

Ask a simple question to PPruners and you get "rocket science answers"... rocket science it aint; but there are many PPruners who wish to go on a "points scoring" trip! However, it would be great if the "point scorers" could write in "correct" grammatical English that everyone can understand.